Вы находитесь на странице: 1из 37

PART III-A

CASE 29: CEBU SEAMENS ASSOCIATION, INC VS


FERRER-CALLEJA (TORRES)
TOPIC: Definitions: Legitimate Workers Association
FACTS: In Oct 1950, a group of deck officers and marine
engineers on board vessels plying Cebu and other ports of the
Philippines organized into an association and registered as a
non-stock corporation known as Cebu Seamens Association,
Inc. with the SEC. The same group registered its association
with the BLR as a labor union known as the Seamens
Association of the Philippines, Inc. (SAPI).
SAPI had an existing CBA with the Aboitiz Shipping Corp. The
company has been remitting checked-off union dues until Feb
1987, when a group composed of the members of the said
union, headed by Manuel Gabayoyo, introduced itself as the
new officers and claimed that they were entitled to such dues.
On May 1987, another group headed by Dominica Nacua filed
a complaint for and on behalf of the union, against CSAI for
the security of aforementioned CBA, seeking to restrain CSAI
from acting on behalf of the union and directing Aboitiz to remit
the checked-off union dues for the months of March and April
1987.
CSAI filed its Answer/Position Paper alleging that the
complainant union and CSAI are one and the same; that
Nacua and an Atty Padilla who represented the union had
been expelled for lawful causes; and that its new set of officers
headed by Gabayoyo has the lawful right to funds. To bolster
its claim, CSAI filed a Motion to Dismiss on the ground that it
was the SEC, not the med-arbiter who had jurisdiction over the

dispute, and that Nacua has no personality to represent the


union as they were already expelled as members/officers of
the union.
The med-arbiter issued an order denying said motion and
directed Aboitiz to remit the checked off union dues to the
complainant union through its officers. CSAI then filed an
appeal with the BLR, but the med-arbiters decision was
affirmed. Another appeal was made to the Secretary of the
Department of Labor which was considered, but was later on
denied for lack of merit.
ISSUE: WON SAPI is a legitimate labor organization, and
therefore entitled to the custody of the union dues
HELD: YES
RATIO: CSAI is not a legitimate labor organization because it
is only registered with SEC. It is the registration of the
organization with the Bureau of Labor Relations and not with
the SEC which made it a legitimate labor organization with
rights and privileges granted under the Labor Code.
On the basis of the evidence presented by the parties, SAPI,
the legitimate labor union, registered with its office, is not the
same association as CSAI, the corporation, insofar as their
rights under the Labor Code are concerned.
Hence, SAPI and not the CSAI is entitled to the release and
custody of union fees with Aboitiz Shipping and other shipping
companies with whom it had an existing CBA. The election of
the so-called set of officers headed by Manuel Gabayoyo was
conducted under the supervision of the SEC. That being the
case, the aforementioned set of officers is of the CSAI and not

LABOR 2 | Atty. Aonuevo | BETT DIGEST GROUP |

of SAPI. It follows, then, that any proceedings, and actions


taken by said set of officers cannot, in any manner, affect the
union and its members.
CASE 30: GUIJARNO VS CIR (TORRES)
TOPIC: Definition; Union Rationale
FACTS: 3 unfair labor practice cases for unlawful dismissal
allegedly based on legitimate union activity were filed against
the respondents Central Santos Lopez Co. and United Sugar
Workers Union-ILO. The expulsion was due to the closed-shop
provision in the then existing Collective Bargaining Contract
(CBC).
The respondent company said that the dismissal was asked by
USWU-ILO in which such union had a CBC with a closed-shop
provision that those laborers who are no longer members of
good standing in the union may be dismissed by the
respondent company if their dismissal is sought by the union;
It never committed acts of unfair labor against the
complainants since it was only complying with the terms and
conditions of the CBC; and that a closed-shop agreement is
expressly allowed under the Industrial Peace Act (RA 875).
The respondent court, in its decision, acknowledged the
existence of such employment relationship. It further
concluded that the dismissal was justifiable under the closedshop provision of the CBC.
ISSUE: WON the closed shop provision can be applied
retroactively.
HELD: NO. The closed shop provision cannot be applied
retroactively.

RATIO: The closed-shop agreement authorized under sec. 4


subsec. a(4) of the Industrial Peace Act should apply to
persons to be hired, or to employees who are not yet members
of any labor organization. It is inapplicable to those who are
members of another union. Employees in a company who are
members of a minority union may be compelled to disaffiliate
from their union and join the majority or contracting union
would render nugatory the right of all employees to selforganization and to form, join or assist labor organizations of
their own choosing.
If said union may be compelled to admit new members, who
have the requisite qualifications, with more reason may the law
and the courts exercise the coercive power when the
employee involved is a long standing union member, who,
owing to provocations of union officers, was impelled to tender
his resignation, which he forthwith withdrew or revoked.
Workers unorganized are weak; workers organized are strong.
Necessarily then, they join labor unions. To further increase
the effectiveness of such organizations, a closed-shop has
been allowed. It could happen though that such stipulation
which assures further weight to a labor union at the bargaining
table could be utilized against minority groups or individual
members. The respondent court, it would appear, was not alert
to such danger.
The dismissed workers should be reinstated to work with back
pay.
CASE 31: PAFLU v. Secretary, TUTAAN
TOPIC: Government Regulation (Definition)

LABOR 2 | Atty. Aonuevo | BETT DIGEST GROUP |

Facts: Petitioners pray for writs of certiorari and prohibition to


restrain respondents, the Secretary of Labor, the Director of
Labor Relations and the Registrar of Labor Organizations,
from enforcing an order of cancellation of the registration
certificate of the Social Security System Employees
Association (SSSEA) which is affiliated to the Philippine
Association of Free Labor Unions (PAFLU). The Registrar of
Labor Organizations issued a notice of hearing, of the matter
of cancellation of the registration of the SSSEA, because of:
(1) Failure to furnish the Bureau of Labor Relations with copies
of the reports on the finances of that union duly verified by
affidavits which its treasurer or treasurers rendered to said
union and its members covering the periods from September
24, 1960 to September 23, 1961 and September 24, 1961 to
September 23, 1962, inclusive, within sixty days of the 2
respective latter dates, which are the end of its fiscal year; and
(2) Failure to submit to this office the names, postal addresses
and non-subversive affidavits of the officers of that union within
sixty days of their election in October (1st Sunday), 1961 and
1963, in conformity with Article IV(1) of its constitution and bylaws in violation of Section 23 of Republic Act No. 875.
Counsel for the SSSEA moved to postpone the hearing, and to
submit then a memorandum, as well as the documents
specified in the notice. The motion was granted, but nobody
appeared for the SSSEA on the date last mentioned. Manuel
Villagracia, Assistant Secretary of the SSSEA, filed with the
Office of the Registrar, the following: (1) Joint non-subversive
affidavit of the officers of the SSS Employees AssociationPAFLU; (2) List of newly-elected officers of the Association in
its general elections held on April 29, 1963; and (3) Copy of
the amended constitution and by-laws of the Association.

Issue: WON the effect of Section 23 of Republic Act No. 875


unduly curtails the freedom of assembly and association
guaranteed in the Bill of Rights?
Ratio: There is no incompatibility between Republic Act No.
875 and the Universal Declaration of Human Rights. Upon the
other hand, the cancellation of the SSSEAs registration
certificate would not entail a dissolution of said association or
its suspension. The existence of the SSSEA would not be
affected by said cancellation, although its juridical personality
and its statutory rights and privileges as distinguished from
those conferred by the Constitution would be suspended
thereby.
To be registered, pursuant to Section 23(b) of Republic Act No.
875, a labor organization, association or union of workers must
file with the Department of Labor the following documents: (1)
A copy of the constitution and by-laws of the organization
together with a list of all officers of the association, their
addresses and the address of the principal office of the
organization; (2) A sworn statement of all the officers of the
said organization, association or union to the effect that they
are not members of the Communist Party and that they are not
members of any organization which teaches the overthrow of
the Government by force or by any illegal or unconstitutional
method; and (3) If the applicant organization has been in
existence for one or more years, a copy of its last annual
financial report. Moreover, paragraph (d) of said-Section
ordains that: The registration and permit of a legitimate labor
organization shall be cancelled by the Department of Labor, if
the Department has reason to believe that the labor
organization no longer meets one or more of the requirements
of paragraph (b) above; or fails to file with the Department
Labor either its financial report within the sixty days of the end

LABOR 2 | Atty. Aonuevo | BETT DIGEST GROUP |

of its fiscal year or the names of its new officers along with
their non-subversive affidavits as outlined in paragraph (b)
above within sixty days of their election; however, the
Department of Labor shall not order the cancellation of the
registration and permit without due notice and hearing, as
provided under paragraph (c) above and the affected labor
organization shall have the same right of appeal to the courts
as previously provided.
CASE 32: San Miguel Corp. Employees Union v. SM
Packing (BUBAN)
TOPIC: Requirements
FACTS: Petitioner(s): the incumbent bargaining agent for the
bargaining unit comprised of the regular monthly-paid rank
and file employees of the three divisions of San Miguel
Corporation (SMC), namely, the San Miguel Corporate Staff
Unit (SMCSU), San Miguel Brewing Philippines (SMBP), and
the San Miguel Packaging Products (SMPP), in all offices and
plants of SMC, including the Metal Closure and Lithography
Plant in Laguna. It had been the certified bargaining agent for
20 years (1987 to 1997).
Respondent(s): SMPPEU-PDMP is registered as a chapter
PDMP
PDMP issued a charter certificate to respondent. In
compliance with registration requirements, respondent
submitted the requisite documents to the BLR for the purpose
of acquiring legal personality.
Upon submission of
documents, respondent
Local or Chapter by the
Arbiter of DOLE-NCR,

its charter certificate and other


was issued Certificate of Creation of
BLR. Respondent filed with the Med3 separate petitions for certification

election to represent SMPP, SMCSU, and SMBP. All 3


petitions were dismissed, on the ground that the separate
petitions fragmented a single bargaining unit.
Petitioner filed with the DOLE-NCR a petition seeking the
cancellation of respondent's registration and its dropping from
the rolls of legitimate labor organizations, accusing respondent
of committing fraud and falsification, and non-compliance with
registration requirements in obtaining its certificate of
registration.o It alleged that respondent violated Articles 239
(a), (b) and (c) and 234 (c) of the Labor Code. Moreover,
petitioner claimed that PDMP is not a legitimate labor
organization, but a trade union center, hence, it cannot directly
create a local or chapter.
DOLE-NCR Regional Director Maximo B. Lim issued an Order
dismissing the allegations. He furtherruled that respondent is
allowed to directly create a local or chapter. However, he found
that respondent did not comply with the 20% membership
requirement and, thus, ordered the cancellation of its
certificate of registration and removal from the rolls of
legitimate labor organizations.
Respondent appealed to the BLR who granted the petition.
The BLR ruled that as a chartered local union, respondent is
not required to submit the number of employees and names of
all its members comprising at least 20% of the employees in
the bargaining unit where it seeks to operate. Thus, the
revocation of its registration based on non-compliance with the
20% membership requirement does not have any basis in the
rules. The BLR also held that although PDMP is considered as
a trade union center, it is a holder of a Registration Certificate
issued by the BLR on 14 February 1991, which bestowed
upon it the status of a legitimate labor organization with all the

LABOR 2 | Atty. Aonuevo | BETT DIGEST GROUP |

rights and privileges to act as representative of its members


for purposes of collective bargaining agreement. On this basis,
PDMP can charter or create a local, in accordance with the
provisions of Department Order No. 9. BLR denied petitioners
appeal.
CA affirmed BLR decision holding that Department Order No. 9
provides that a registered federation or national union may
directly create a local by submitting to the BLR copies of the
charter certificate, the local's constitution and by-laws, the
principal office address of the local, and the names of its
officers and their addresses. Upon complying with the
documentary requirements, the local shall be issued a
certificate and included in the roster of legitimate labor
organizations. Thus there is no need for SMPPEU to show a
membership of 20% of the employees of the bargaining unit in
order to be recognized as a legitimate labor union.
ISSUE: WON respondent is a legitimate labor organization
even if it failed to comply with the 20% requirement as
provided in Art. 234, LC. NO
HELD: NO
RATIO: A legitimate labor organization is defined as "any labor
organization duly registered with the DOLE, and includes any
branch or local thereof."
Why does the Labor Code demand strict compliance with the
requirements on registration? Registration requirements are
intended to afford a measure of protection to unsuspecting
employees who may be lured into joining unscrupulous or flyby-night unions whose sole purpose is to control union fundsor
use the labor organization for illegitimate ends. A legitimate

labor organization is entitled to specific rights under the Labor


Code, 21 and are involved in activities directly affecting
matters of public interest. Legitimate labor organizations have
exclusive rights under the law which cannot be exercised by
non-legitimate unions, one of which is the right to be certified
as the exclusive representative of all the employees in an
appropriate collective bargaining unit for purposes of collective
bargaining.
The acquisition of rights by any union or labor organization,
particularly the right to file a petition for certification election,
first and foremost, depends on whether or not the labor
organization has attained thestatus of a legitimate labor
organization. Records show that respondent was chartered by
PDMP. Article 234, LC provides that an independent labor
organization acquires legitimacy only upon its registration with
the BLR. However, the creation of a branch, local or chapter is
treated differently.
In Progressive Development Corporation v. Secretary,
Department of Labor and Employment, the Court declared that
when an unregistered union becomes a branch, local or
chapter, some of the aforementioned requirements for
registration are no longer necessary or compulsory. Whereas
an applicant for registration of an independent union is
mandated to submit, among other things, the number of
employees and names of all its members comprising at least
20% of the employees in the bargaining unit where it seeks to
operate, as provided under Article 234 and Sec. 2, Rule III,
Book V of the Implementing Rules, the same is no longer
required of a branch, local or chapter.
The intent of the law in imposing less requirements in the case
of a branch or local of a registered federation or national union

LABOR 2 | Atty. Aonuevo | BETT DIGEST GROUP |

is to encourage the affiliation of a local union with a federation


or national union in order to increase the local union's
bargaining powers respecting terms and conditions of labor.
Petitioners argue that PDMP is not a legitimate labor
organization, thus cannot form a charter.
The Court held that the personality of a labor organization
cannot be attacked collaterally. It may be questioned only in an
independent petition for cancellation in accordance with
Section 5 of Rule V, Book V of the Implementing Rules.
TWIST: PDMP is a trade union center, THEREFORE IT
CANNOT CREATE LOCALS OR CHARTERS. Trade union
center was never mentioned in the Labor Code. It first
appeared only in the Implementing Rules of Department Order
No. 9 which defined a trade union center as any group of
registered national unions or federations organized for the
mutual aid and protection of its members; for assisting such
members in collective bargaining; or for participating in the
formulation of social and employment policies, standards, and
programs, andis duly registered with the DOLE in accordance
with Rule III, Section 2 of the Implementing Rules.
While a "national union" or "federation" is a labor organization
with at least ten locals or chapters or affiliates, each of which
must be a duly certified or recognized collective bargaining
agent, a trade union center, on the other hand, is composed of
a group of registered national unions or federations. The
Implementing Rules, as amended by Department Order No. 9,
provide that only "a duly registered federation or national
union" may directly create a local or chapter. DO 9 defines a
"chartered local" as a labor organization in the private sector
operating at the enterprise level that acquired legal personality
through a charter certificate, issued by a duly registered

federation or national union and reported to the Regional


Office in accordance with Rule III, Section 2-E of these Rules
(Sec. 1 (i), Rule 1, Book V of the Implementing Rules, as
amended by DO No. 9)
RA 9481 or "An Act Strengthening the Workers' Constitutional
Right to Self-Organization, Amending for the Purpose
Presidential Decree No. 442, As Amended, Otherwise Known
as the Labor Code of the Philippines" lapsed into law on 25
May 2007 and became effective on 14 June 2007. This law
further amends the Labor Code provisions on Labor Relations,
including trade union centers in Art. 234. However, it still
makes no mention that such organizations can create a local
or a charter.
Therefore, since under the pertinent status and applicable
implementing rules, the power granted to labor organizations
to directly create a chapter or local through chartering is given
to a federation or national union, then a trade union center is
without authority to charter directly. This is to prevent
circumvention of labor union requirements. As a legitimate
labor organization is entitled to specific rights under the Labor
Code and involved in activities directly affecting public interest,
it is necessary that the law afford utmost protection to the
parties affected.
CASE 33: PROGRESSIVE DEVELOPMENT CORP. VS
SECRETARY OF LABOR
TOPIC: Government Regulation; Union Registration;
Requirements
FACTS: Respondent Pambansang Kilusan ng Paggawa
(Kilusan) TUCP filed with the DOLE a petition for certification
of election among the local rank-and-file employees of the

LABOR 2 | Atty. Aonuevo | BETT DIGEST GROUP |

petitioner alleging that it is a legitimate labor federation and its


local chapter, Progressive Development Employees Union
(PDEU) was issued a charter certificate. Kilusan alleged that
there was no existing collective bargaining agreement and that
no other legitimate labor organization existed in the bargaining
unit. PDC filed a motion to dismiss the certification election
arguing that PDEU has not yet submitted the required
documents in order to be eligible for a certification election
(Constitution and by-laws, names, addresses and list of
officers and/or members, and books of account). Kilusan
submitted a rejoinder to the motion to dismiss filed arguing that
it had already submitted the above-mentioned documents
except the books of account as PDEU has just been recently
recognized and no books of account are available yet.
Petitioner insisted that upon verification with the Bureau of
Labor Relations (BLR), it found that the alleged minutes of the
organizational meeting was unauthenticated, the list of
members did not bear the corresponding signatures of the
purported members, and the constitution and by-laws did not
bear the signature of the members and was not duly
subscribed. It argued that the private respondent therefore
failed to substantially comply with the registration requirements
provided by the rules. Med-Arbiter dela Cruz found that there
was substantial compliance with the documentary
requirements and resolved in favor of a certification of election.
Petitioner filed its motion for reconsideration with the Secretary
of Labor, which was treated as an appeal. The Secretary of
Labor dismissed the MR filed by petitioner, hence this present
appeal.
ISSUE: W/N respondent has complied with the requirements
for PDEUs legitimacy?
HELD: NO, the statutory requirements have not been satisfied.

RATIO: Ordinarily, a labor organization acquires legitimacy


only upon registration with the BLR. Under Article 234
(Requirements of Registration):
Any applicant labor organization, association or group of
unions or workers shall acquire legal personality and shall be
entitled to the rights and privileges granted by law to legitimate
labor organizations upon issuance of the certificate of
registration based on the following requirements:
(a) Fifty-pesos (P50.00) registration fee;
(b) The names of its officers, their addresses, the principal
address of the labor organization, the minutes of the
organizational meeting and the list of the workers who
participated in such meetings;
(c) The names of all its members comprising at least twenty
20% percent of all the employees in the bargaining unit where
it seek to operate;
(d) If the applicant has been in existence for one or more
years, copies , of its annual financial reports; and
(e) Four copies of the constitution and by-laws of the applicant
union, the minutes of its adoption or ratification and the list of
the members who participated in it.
And under Article 235:
The Bureau shall act on all applications for registration within
thirty (30) days from filing.
All requisite documents and papers shall be certified under
oath by the secretary or the treasurer of the organization, as
the case may be, and attested to by its president.
In the case at bar, the constitution and by-laws and list of
officers submitted in the BLR, while attested to by the
chapter's president, were not certified under oath by the
secretary.

LABOR 2 | Atty. Aonuevo | BETT DIGEST GROUP |

In the case of union registration, the rationale for requiring that


the submitted documents and papers be certified under oath
by the secretary or treasurer, as the case may be, and attested
to by president is apparent. The submission of the required
documents (and payment of P50.00 registration fee) becomes
the Bureau's basis for approval of the application for
registration. Upon approval, the labor union acquires legal
personality and is entitled to all the rights and privileges
granted by law to a legitimate labor organization. The
employer naturally needs assurance that the union it is dealing
with is a bona fide organization, one which has not submitted
false statements or misrepresentations to the Bureau.
The certification and attestation requirements are preventive
measures against the commission of fraud. They likewise
afford a measure of protection to unsuspecting employees who
may be lured into joining unscrupulous or fly-by-night unions
whose sole purpose is to control union funds or to use the
union for dubious ends.
A local or chapter therefore becomes a legitimate labor
organization only upon submission of the following to the BLR:
1) A charter certificate, within 30 days from its issuance by the
labor federation or national union, and
2) The constitution and by-laws, a statement on the set of
officers, and the books of accounts all of which are certified
under oath by the secretary or treasurer, as the case may be,
of such local or chapter, and attested to by its president.
Absent compliance with these mandatory requirements, the
local or chapter does not become a legitimate labor
organization.

The failure of the secretary of PDEU-Kilusan to certify the


required documents under oath is fatal to its acquisition of a
legitimate status.
CASE 34: Takata (Philippines) Corporation v. BLR, G.R.
No. 196276, June 4, 2014 TORRES
TOPIC 1: Government Regulation; Union Registration;
Requirements
TOPIC 2: Government Regulation; Cancellation of Union
Certificate of Registration
FACTS: On July 7, 2009, petitioner filed with DOLE Regional
Office a Petition for Cancellation of the Certificate of Union
Registration of Respondent, Samahang Lakas Manggagawa
ng Takata (SALAMAT), on the ground that the latter is guilty of
misrepresentation, false statement and fraud with respect to
the number of those who participated in the organizational
meeting, the adoption and ratification of its Constitution and
By-Laws, and in the election of its officers.
It contended that in the May 1, 2009 meeting of respondent,
only 68 attendees signed the attendance sheet, which is only
17% of the 396 regular employees and hence, respondent
failed to comply with the 20% minimum membership
requirement. Petitioner insisted that the employees were not
given sufficient information when they signed the document
Sama-samang Pahayag ng Pagsapi, and that the document
Pangalan ng mga Kasapi ng Unyon bore no signatures of the
alleged 119 members; that the union members were only 117;
and that the total number of petitioners employees as of May
1, 2009 were 470.
The respondent denied the charge and claimed that the 119
union members were more than the 20% requirement, and it
was supported by the document Sama-samang Pahayag ng

LABOR 2 | Atty. Aonuevo | BETT DIGEST GROUP |

Pagsapi. It also contended that the petitioner is stopped from


assailing its legal personality as it agreed to a certification
election and actively participated in the pre-election
conference. The union members were informed of the contents
of the documents they signed, and that the 68 attendees
constituted more than 50% of the total union membership,
hence, a quorum existed.
DOLE Reg. Dir. Atty. Martinez issued a decision granting the
petition for cancellation of the respondents certificate of
registration. The respondent then filed a Notice of
Memorandum of Appeal with the BLR, first through BMP
Paralegal Officer Mole, then through its counsels, Attys.
Banzuela and Velandrez accompanied by a Formal Entry of
Appearance. Petitioner opposed the appeal due to forum
shopping; failing to avail of the correct remedy within the
period; and that certificate of registration was tainted with
fraud, misrepresentation and falsification.
The BLR then reversed the decision of the regional director
since the petitioner failed to prove that the respondent
deliberately and maliciously misrepresented the number of
employees. The BLR also found that the list of employees who
participated in the organizational meeting was a separate and
distinct requirement from the list of names of members
comprising at least 20% of the employees in the bargaining
unit.
The petitioner filed a Petition for Certiorari with the CA, but the
CA denied the petition and affirmed the decision of the BLR.
ISSUES: WON the CA committed error:
1) In affirming the decision of the BLR in not finding that the
respondent SALAMAT violated the rule on forum shopping.

2) In finding that the application for registration of SALAMAT


was compliant with the law.
HELD:
1) NO. The BLR did not commit error in finding that SALAMAT
did not violate the rule on forum shopping.
2) NO. The application for registration of SALAMAT was
compliant with the law.
RATIO: 1) It is undisputed that BMP was no longer authorized
to file an appeal since its services was already terminated.
Since the appeal filed with the BLR was not specifically
authorized by the respondent, such appeal is considered to
not have been filed at all. An unauthorized complaint does not
produce any legal effect.
2) The 20% minimum requirement pertains to the employees
membership in the union and not to the list of workers who
participated in the organizational meeting. Indeed, Article
234(b) and (c) provide for separate requirements, which must
be submitted for the unions registration, and which respondent
did submit. Here, the total number of employees in the
bargaining unit was 396, and 20% of which was about 79.
Respondent submitted a document entitled Pangalan ng Mga
Kasapi ng Unyon showing the names of 119 employees as
union members, thus respondent sufficiently complied even
beyond the 20% minimum membership requirement.
Respondent also submitted the attendance sheet of the
organizational meeting which contained the names and
signatures of the 68 union members who attended the
meeting. Considering that there are 119 union members which
are more than 20% of all the employees of the bargaining unit,
and since the law does not provide for the required number of
members to attend the organizational meeting, the 68

LABOR 2 | Atty. Aonuevo | BETT DIGEST GROUP |

attendees which comprised at least the majority of the 119


union members would already constitute a quorum for the
meeting to proceed and to validly ratify the Constitution and
By-laws of the union.
Although there was an employee whose name appeared twice
and another who was merely a project employee, the court
said that For fraud and misrepresentation to be grounds for
cancellation of union registration under the Labor Code, the
nature of the fraud and misrepresentation must be grave and
compelling enough to vitiate the consent of a majority of union
members.
CASE 35: Legend International Resorts Limited v.
Kilusang Manggagawa ng Legenda, G.R. No.169754,
February 23, 2011 TUTAAN
TOPIC: EFFECT OF REGULATION
Facts: KML filed with the Med-Arbitration Unit of the DOLE,
San Fernando, Pampanga, a Petition for Certification Election.
KML alleged that it is a legitimate labor organization of the
rank and file employees of Legend International Resorts
Limited (LEGEND). KML claimed that it was issued its
Certificate of Registration by the DOLE on May 18, 2001.
LEGEND moved to dismiss the petition alleging that KML is
not a legitimate labor organization because its membership is
a mixture of rank and file and supervisory employees in
violation of Article 245 of the Labor Code. LEGEND also
claimed that KML committed acts of fraud and
misrepresentation when it made it appear that certain
employees attended its general membership meeting when in
reality some of them were either at work; have already
resigned; or were abroad. In its Comment, KML argued that
even if 41 of its members are indeed supervisory employees

and therefore excluded from its membership, the certification


election could still proceed because the required number of
the total rank and file employees necessary for certification
purposes is still sustained. KML also claimed that its legitimacy
as a labor union could not be collaterally attacked in the
certification election proceedings but only through a separate
and independent action for cancellation of union registration.
Finally, as to the alleged acts of misrepresentation, KML
asserted that LEGEND failed to substantiate its claim.
Ruling of the Med-Arbiter: The Med-Arbiter rendered judgment
dismissing for lack of merit the petition for certification election.
It found that indeed there were several supervisory employees
in KMLs membership. Since Article 245 of the Labor Code
expressly prohibits supervisory employees from joining the
union of rank and file employees, the Med-Arbiter concluded
that KML is not a legitimate labor organization. KML was also
found to have fraudulently procured its registration certificate
by misrepresenting that 70 employees were among those who
attended its organizational meeting when in fact they were
either at work or elsewhere. KML thus appealed to the Office
of the Secretary of the DOLE.
Ruling of the Office of the Secretary of DOLE: The Office of
the Secretary of DOLE rendered its Decision granting KMLs
appeal thereby reversing and setting aside the Med-Arbiters
Decision. It held that KMLs legitimacy as a union could not be
collaterally attacked, citing Section 5, Rule V of Department
Order No. 9, series of 1997. The Office of the Secretary of
DOLE also opined that Article 245 of the Labor Code merely
provides for the prohibition on managerial employees to form
or join a union and the ineligibility of supervisors to join the
union of the rank and file employees and vice versa. It
declared that any violation of the provision of Article 245 does

LABOR 2 | Atty. Aonuevo | BETT DIGEST GROUP |

10

not ipso facto render the existence of the labor organization


illegal. Moreover, it held that Section 11, paragraph II of Rule
XI which provides for the grounds for dismissal of a petition for
certification election does not include mixed membership in
one union. LEGEND filed its Motion for Reconsideration
reiterating its earlier arguments. It also alleged that it filed a
Petition for Cancellation of Union Registration of KML which
was granted by the DOLE Regional Office No. III of San
Fernando, Pampanga. The Office of the Secretary of DOLE
denied LEGENDs motion for reconsideration. It opined that
Section 11, paragraph II(a), Rule XI of Department Order No. 9
requires a final order of cancellation before a petition for
certification election may be dismissed on the ground of lack of
legal personality.
Ruling of the Court of Appeals: Undeterred, LEGEND filed a
Petition for Certiorari with the CA. LEGEND alleged that the
Office of the Secretary of DOLE gravely abused its discretion
in reversing and setting aside the Decision of the Med-Arbiter
despite substantial and overwhelming evidence against KML.
For its part, KML alleged that the Decision of the Bureau of
Labor Relations in denying LEGENDs petition for cancellation
and upholding KMLs legitimacy as a labor organization has
already become final and executory. The Office of the
Secretary of DOLE also filed its Comment asserting that KMLs
legitimacy cannot be attacked collaterally. Finally, the Office of
the Secretary of DOLE stressed that LEGEND has no legal
personality to participate in the certification election
proceedings. The CA rendered its Decision finding no grave
abuse of discretion on the part of the Office of the Secretary of
DOLE. The appellate court held that the issue on the
legitimacy of KML as a labor organization has already been
settled with finality. The Decision of the Bureau of Labor
Relations upholding the legitimacy of KML as a labor

organization had long become final and executory for failure of


LEGEND to appeal the same. Thus, having already been
settled that KML is a legitimate labor organization, the latter
could properly file a petition for certification election. There
was nothing left for the Office of the Secretary of DOLE to do
but to order the holding of such certification election.
Issue: WON the cancellation of KMLs certificate of registration
should retroact to the time of its issuance? NO
WON the legal personality of KML can be collaterally attacked
in a petition for certification election? NO
Ratio: Petitioners Arguments: LEGEND submits that the
Court of Appeals grievously erred in denying its Petition for
Cancellation of KMLs registration has already become final
and executory. It asserts that it has seasonably filed a Petition
for Certiorari before the CA assailing said Decision. In fact, the
CA granted the petition, reversed the March 26, 2002 Decision
of the Bureau of Labor Relations and reinstated the November
7, 2001 Decision of the DOLE Regional Office III ordering the
cancellation of KMLs registration. Finally, LEGEND posits that
the cancellation of KMLs certificate of registration should
retroact to the time of its issuance. It thus claims that the
petition for certification election and all of KMLs activities
should be nullified because it has no legal personality to file
the same, much less demand collective bargaining with
LEGEND. LEGEND thus prays that the Decision of the MedArbiter dismissing KMLs petition for certification election be
reinstated.
Respondents Arguments: In its Comment filed before this
Court, KML insists that the Decision of the Bureau of Labor
Relations upholding its legitimacy as a labor organization has

LABOR 2 | Atty. Aonuevo | BETT DIGEST GROUP |

11

already attained finality hence there was no more hindrance to


the holding of a certification election. Moreover, it claims that
the instant petition has become moot because the certification
election sought to be prevented had already been conducted.
This issue is not new or novel. In Pepsi-Cola Products
Philippines, Inc. v. Secretary of Labor, we already ruled that:
Anent the issue of whether or not the Petition to cancel/revoke
registration is a prejudicial question to the petition for
certification election, the following ruling in the case of
Association of the Court of Appeals Employees (ACAE) v. Hon.
Pura Ferrer-Calleja is in point, to wit: At any rate, the Court
applies the established rule correctly followed by the public
respondent that an order to hold a certification election is
proper despite the pendency of the petition for
cancellation of the registration certificate of the
respondent union. The rationale for this is that at the time the
respondent union filed its petition, it still had the legal
personality to perform such act absent an order directing the
cancellation. In Capitol Medical Center, Inc. v. Hon. Trajano,
we also held that the pendency of a petition for cancellation
of union registration does not preclude collective
bargaining. Citing the Secretary of Labor, we held viz.: That
there is a pending cancellation proceedings against the
respondent Union is not a bar to set in motion the mechanics
of collective bargaining. If a certification election may still
be ordered despite the pendency of a petition to cancel
the unions registration certificate more so should the
collective bargaining process continue despite its
pendency. We agree with the ruling of the Office of the
Secretary of DOLE that the legitimacy of the legal personality
of KML cannot be collaterally attacked in a petition for
certification election proceeding. This is in consonance with
our ruling in Laguna Auto parts, Manufacturing Corporation v.

Office of the Secretary, Department of Labor and Employment,


that such legal personality may not be subject to a
collateral attack but only through a separate action
instituted particularly for the purpose of assailing it. The
legal personality of a legitimate labor organization cannot be
subject to a collateral attack. The law is very clear on this
matter. The Implementing Rules stipulate that a labor
organization shall be deemed registered and vested with
legal personality on the date of issuance of its certificate
of registration. Once a certificate of registration is issued
to a union, its legal personality cannot be subject to a
collateral attack. In may be questioned only in an
independent petition for cancellation in accordance with
Section 5 of Rule V, Book V of the Implementing Rules.
CASE 36: Acedera v. International Container Terminal
Services, Inc., (BUBAN)
TOPIC: Rights of Legitimate Labor Organization
FACTS: Ordinarily, a person whose interests are already
represented will not be permitted todo the same except when
there is a suggestionof fraud or collusion or that
therepresentative will not act in good faith.
Jerry Acedera, et al. are employees of International Container
Terminal Services, Inc. (ICTSI) and are members of
Associated
Port
Checkers
&
Workers
UnionInternationalContainer Terminal Services, Inc. (APCWUICTSI), a duly registered labor organization. ICTSI entered into
a five-year Collective BargainingAgreement (CBA) with
APCWU which reduced the employees work days from 304 to
250 days a year.
The Wage Board decreed wage increases in NCR which
affected ICTSI. Upon the request of APCWU to compute the

LABOR 2 | Atty. Aonuevo | BETT DIGEST GROUP |

12

actual monthly increase in the employees salary by multiplying


the mandated increase by 365 days and dividing by 12
months, ICTSI stopped using 304 days as divisor and started
using 365 days to determine the dailywage.

may act as therepresentative of its members for the purpose of


CBA. This authority includes the power torepresent its
members for the purpose of enforcing the provisions of the
CBA.

Later on, ICTSI entered into a retrenchment program which


prompted APCWU to file acomplaint before the Labor Arbiter
(LA) for ICTSIs use of 365 days, instead of 250 days, as
divisor in the computation of wages. Acedera et al. filed a
Motion to Intervene which was denied by the LA. On appeal,
NLRC affirmed LAs decision. Acedera et al. filed a petition
forcertiorari to the Courtof Appeals (CA) which was dismissed.

That APCWU acted in a representative capacity "for and in


behalf of its Unionmembers and other employees similarly
situated, the title of the case filed by it at the LaborArbiters
Office so expressly states.While a party acting in a
representative capacity, such as a union, may be permitted
tointervene in a case, ordinarily, a person whose interests are
already represented will not bepermitted to do the same
except when there is a suggestion of fraud or collusion or that
therepresentative will not act in good faith for the protection of
all interests represented byhim.

ISSUE: WON Acedera et al. have no legal right to intervene in


the case as their intervention was asuperfluity
HELD: Yes, they have no right.
RATIO: Acedera et al. stress that they have complied with the
requisites for intervention because (1) they are the ones who
stand to gain or lose by the direct legal operation andeffect of
any judgment that may be rendered in this case, (2) no undue
delay or prejudicewould result from their intervention since
their Complaint-in-Intervention with Motion forIntervention was
filed while the Labor Arbiter was still hearing the case and
before anydecision thereon was rendered, and (3) it was not
possible for them to file a separate caseas they would be guilty
of forum shopping because the only forum available for them
wasthe Labor Arbiter.
Acedera et al., however, failed to consider, in addition to the
rule on intervention, therule on representation. A labor union is
one such party authorized to represent its membersunder
Article 242(a) of the Labor Code which provides that a union

Acedera et al. cite the dismissal of the case filed by ICTSI, first
by the Labor Arbiter,and later by the Court of Appeals. The
dismissal of the case does not, however, by itself show the
existence of fraud or collusion or a lack of good faith on the
part of APCWU. There must be clear and convincing evidence
of fraud or collusion or lack of good faithindependently of the
dismissal. This, Acedera et al. failed to proffer. Acedera et al.
likewise express their fear that APCWU would not prosecute
the casediligently because of its sweetheart relationship" with
ICTSI. There is nothing on record,however, to support this
alleged relationship which allegation surfaces as a mere
afterthought because it was never raised early on. It was
raised only in petitioners-appellants reply to ICTSIs comment
in the petition at bar, the last pleading submitted tothis Court,
which was filed on June 20, 2001 or more than 42 months
after petitioners-Appellants filed their Complaint-in-Intervention
with Motion to Intervene with theLabor Arbiter. To reiterate, for
a member of a class to be permitted to intervene in

LABOR 2 | Atty. Aonuevo | BETT DIGEST GROUP |

13

arepresentative action, fraud or collusion or lack of good faith


on the part of the representative must be proven. It must be
based on facts borne on record. Mere assertions,as what
petitioners-appellants proffer, do not suffice.
CASE 37: Tropical Hut Employees Union v. Tropical Hut
Food Market, Inc., 181 SCRA 173 (1990) ESTOMO
Topic: Effects of Non-Registration
Facts: The rank-and-file employees of Tropical Hut Food
Market Inc. organized a local union called Tropical Hut
Employees Union (THEU) and affiliated itself with the National
Association of Trade Unions (NATU), a non-registered labor
organization. NATU accepted the affiliation and THEU-NATU
became the sole bargaining representative of the the rankand-file employees within respondent corporation. A CBA with
a closed-shop provision was entered into between THEUNATU and respondent. Upon expiration, another CBA was
entered into between the two parties, incorporating the closedshop provision. Then President of THEU-NATU, Dilag, was
promoted to a supervisorial position and he tendered his
resignation as President of THEU-NATU in light of his
promotion. Encinas replaced him as the new President of
THEU-NATU and it is at this point when Encinas, with the
concurrence of the majority of the members of THEU-NATU,
informed NATU of their desire to disaffiliate. THEU then sent a
letter to Confederation of General Workers (CGW), informing
the latter of the formers desire to affiliate itself with the latter.
THEU then became known as THEU-CGW and the newly
formed union demanded from respondent company that it turn
over all the dues that it collected from the said Union.
Respondent company however denied the said request and
then Acting Chairman of THEU-NATU, Lontok, sent a letter to
Encinas informing him that he will be dismissed from service

for violating the closed-shop provision of the then existing


CBA. Furthermore, Lontok informed Encinas that THEUs
disaffiliation with NATU had no basis, as NATUs Constitution
requires at least three months notice before an organization
may disaffiliate itself with NATU. Lontok then sent a letter to
respondent to dismiss Encinas as well as all the other
members of THEU who disaffiliated themselves with THEUNATU. The respondent dismissed the said employees and the
petitioner filed a complaint for unfair labor practices against
THEU-NATU and respondent. The Arbiter ruled in favor of
petitioner, but on appeal to the NLRC, the NLRC reversed the
decision of the Arbiter. Petitioner then appealed the decision of
the NLRC to the Sec of Labor, which affirmed the decision of
the NLRC.
Issue: W/N the dismissal of petitioners members was proper?
Held: No, there is nothing in NATUs Constitution that says
THEU cannot disaffiliate itself from the former.
Ratio: The right of a local union to disaffiliate from its mother
federation is well settled. A local union, being a separate and
voluntary association, is free to serve the interest of all its
members including the freedom to disaffiliate when
circumstances warrant. The inclusion of the word NATU after
the name of the local union THEU in the registration with the
Department of Labor is merely to stress that the THEU is
NATU's affiliate at the time of the registration. It does not mean
that the said local union cannot stand on its own. Neither can it
be interpreted to mean that it cannot pursue its own interests
independently of the federation. When the local union
withdrew from the old federation to join a new federation, it
was merely exercising its primary right to labor organization for
the effective enhancement and protection of common

LABOR 2 | Atty. Aonuevo | BETT DIGEST GROUP |

14

interests. In the absence of enforceable provisions in the


federation's constitution preventing disaffiliation of a local
union a local may sever its relationship with its parent. There is
nothing in the constitution of the NATU or in the constitution of
the THEU-NATU that the THEU was expressly forbidden to
disaffiliate from the federation. The alleged non-compliance of
the local union with the provision in the NATU Constitution
requiring the service of three months notice of intention to
withdraw did not produce the effect of nullifying the
disaffiliation for the following grounds: firstly, NATU was not
even a legitimate labor organization, it appearing that it was
not registered at that time with the Department of Labor, and
therefore did not possess and acquire, in the first place, the
legal personality to enforce its constitution and laws, much
less the right and privilege under the Labor Code to organize
and affiliate chapters or locals within its group, and secondly,
the act of non-compliance with the procedure on withdrawal is
premised on purely technical grounds which cannot rise above
the fundamental right of self-organization.
CASE 38: Air Phil. Corp. v. BLR
Author: TORRES
TOPIC: Government Regulation; Cancellation of Union
Certificate of Registration
FACTS: On March 17, 1999, Air Philippines Flight Attendants
Association (AP-FLAA) filed a petition for certification election
as the sole bargaining representative of the flight attendants of
Air Philippines Corporation (APC). The med-arbiter rendered a
ruling ordering the holding of the certification election on
August 5, 1999 where majority of the votes were cast in favor
of APFLAA.

APC then filed a Petition for Decertification and Cancellation of


Union Registration against APFLAA, alleging that the union
could not be registered as a labor organization because it is
composed of a mixture of supervisory and rank-and-file flight
attendants. According to APC, the position of Lead Cabin
Attendant was supervisory in character, and there were
members of APFLAA who held such position.
On July 18, 2001, DOLE-NCR Reg. Dir. Maraan dismissed the
petition. In his decision, he said that although Art 245 of the
Labor Code states that supervisory employees are not eligible
for membership in labor organizations of rank-and-file
employees, it does not provide a ground for cancellation of
union registration.
APC filed a Motion for Reconsideration/Appeal with the BLR,
but such was denied. APC then filed a Petition for Certiorari
with the CA, but the petition was dismissed outright since APC
failed to avail of the remedy of a prior Motion for
Reconsideration before filing the certiorari petition. APC filed a
Motion for Reconsideration with the CA but this too was denied
for failing to contain proof of service or registry return receipts
to the respondents.
ISSUE: WON APFLAAs union registration may be cancelled
considering that the union is allegedly composed of a mixture
of supervisory and rank-and-file employees.
HELD: NO. It should not be cancelled due to such grounds.

LABOR 2 | Atty. Aonuevo | BETT DIGEST GROUP |

15

RATIO: Article 2451 of the Labor Code, the legal basis for the
petition for cancellation, merely prescribed the requirements
for eligibility in joining a union and did not prescribe the
grounds for cancellation of union registration. For the purpose
of de-certifying a union, it is not enough to establish that the
rank-and-file union includes ineligible employees in its
membership. Pursuant to Article 2392 (a) and (c) of the Labor
Code, it must be shown that there was misrepresentation,
false statement or fraud in connection with the adoption or
ratification of the constitution and bylaws or amendments
thereto, the minutes of ratification, or in connection with the
election of officers, minutes of the election of officers, the list of
voters, or failure to submit these documents together with the
list of the newly elected-appointed officers and their postal
addresses to the BLR.
APC merely argued that APFLAA was not qualified to become
a legitimate labor organization by reason of its mixed
composition of rank-and-file and supervisory employees; and

1 Art. 245. Ineligibility of managerial employees to join any labor organization; right of
supervisory employees.Managerial employees are not eligible to join, assist or form
any labor organization. Supervisory employees shall not be eligible for membership in
a labor organization of the rank-and-file employees but may join, assist or form
separate labor organizations of their own.

2 Art. 239, Labor Code, states: The following shall constitute grounds for cancellation
of union registration:
1. (a) Misrepresentation, false statement or fraud in connection with the adoption or
ratification of the constitution and by-laws or amendments thereto, the minutes of
ratification;
(c) Misrepresentation, false statements or fraud in connection with the election of
officers, minutes of the election of officers, the list of voters, or failure to submit these
documents together with the list of the newly elected-appointed officers and their
postal addresses within thirty (30) days from election.

that APFLAA committed misrepresentation by making it appear


that its composition was composed purely of rank-and-file
employees. Such misrepresentation (if it can be called as
such) as alleged by APC, is not conformable to Article 239 (a)
and (c) of the Labor Code.
CASE 39: S.S. Ventures International, Inc. v. S.S. Ventures
Labor Union, G.R. No. 161690, July 23, 2008; TUTAAN
Facts: SS Ventures International Inc. manufactures sports
shoes. SS Ventures Labor Union is a labor organization
registered with DOLE. The Union filed a petition for
certification election in behalf of its rank and file employees,
supported by 542 signatures, but 82 of which belong to
employees already terminated by Ventures.
Ventures filed a petition to cancel Unions registration, alleging
the following: (a) that the Union forged 82 signatures of its
FORMER employees to make it appear they took part in the
adoption and ratification of the unions constitution; (b) that the
signature of 3 persons were maliciously entered twice; (c) that
no organizational meeting actually took place; and (d) that the
Unions application was not supported by at least 20% rank
and file employees of Ventures (computed after deducting the
signatures of 82 former employees).
The Union denied the forgery of the signatures of 82 former
employees and alleged that:
(a) there was an actual
organizational meeting which took place at the Shoe City
basketball court in Mariveles; (b) that the 82 former employees
were still qualified because the one-year period to question
their dismissal had not yet lapsed; (c) that it had complied with
the 20%-member registration requirement; (d) that the double
signatures were inadvertent human error. Subsequently,
Ventures made supplemental reply claiming that the 82 former
employees were deceived and harassed to signing their

LABOR 2 | Atty. Aonuevo | BETT DIGEST GROUP |

16

attendance in the organizational meeting as supported by the


attached affidavits. The Regional Director of DOLE resolved to
CANCEL Unions certificate of registration.
A motion for reconsideration was forwarded to Bureau of Labor
Relations, which the Bureau treated as an appeal. The
Bureau granted the appeal and REVERSED the decision of
DOLE. The Bureau found that even subtracting the 82
employees, still the number of members is within the 20%
requirement. Therefore, the Union remains to be a legitimate
labor organization. Likewise, the CA dismissed the appeal of
Ventures. Hence this petition.
Issue: WON the Unions certificate of registration be cancelled
on the ground of fraud or the non-compliance with the 20%
membership requirement? NO.
Ratio: The constitutional right to form, join, or assist a union
shall not be abridged. Once registered with the DOLE, a union
is considered a legitimate labor organization endowed with the
right and privileges granted by law to such organization. But
while a certificate of registration confers a union with
legitimacy with the concomitant right to participate in or ask for
certification election in a bargaining unit, the registration may
be canceled or the union may be decertified as the bargaining
unit, in which case the union is divested of the status of a
legitimate labor organization.
Among the grounds for
cancellation is the commission of any of the acts enumerated
in Art. 239(a) of the Labor Code, such as fraud and
misrepresentation in connection with the adoption or
ratification of the unions constitution and like documents.
It is not enough to show that the union includes ineligible
employees in its membership. It must also be shown that there

was misrepresentation, false statement, or fraud in connection


with the application for registration and the supporting
documents, such as the adoption or ratification of the
constitution and by-laws or amendments thereto and the
minutes of ratification of the constitution or by-laws, among
other documents.
Both the BLR and the CA found no fraud or misrepresentation
on the part of the Union. The affidavits secured by Ventures
were mostly undated and submitted by Ventures only after 7
months past filing its petition to cancel. Previous rulings of the
court held that the employees withdrawal from a labor union
made before the filing of the petition for certification election is
presumed voluntary, while withdrawal after the filing of such
petition is considered to be involuntary and does not affect the
same.
After a labor organization has filed the necessary registration
documents, it becomes mandatory for the BLR to check if the
requirements under Art. 234 of the Labor Code have been
sedulously complied with. If the unions application is infected
by falsification and like serious irregularities, especially those
appearing on the face of the application and its attachments, a
union should be denied recognition as a legitimate labor
organization. The approval of Unions Certificate of
Registration implies that its application for registration and the
supporting documents thereof are prima facie free from any
vitiating irregularities.
The Court need not delve into the question of whether these
82 dismissed individuals were still Union members qualified to
vote and affix their signature on its application for registration
and supporting documents. The procedure for acquiring or
losing union membership and the determination of who are

LABOR 2 | Atty. Aonuevo | BETT DIGEST GROUP |

17

qualified or disqualified to be members are matters internal to


the union and flow from its right to self-organization.
Whatever misgivings the petitioner may have with regard to
the 82 dismissed employees is better addressed in the
inclusion-exclusion proceedings during a pre-election
conference. The issue surrounding the involvement of the 82
employees is a matter of membership or voter eligibility. It is
not a ground to cancel union registration. Ventures should not
interfere in the certification election by actively and persistently
opposing the certification election of the Union. A certification
election is exclusively the concern of employees and the
employer lacks the legal personality to challenge it. In fact,
jurisprudence frowns on the employers interference in a
certification election for such interference unduly creates the
impression that it intends to establish a company union.
CASE 40: Mariwasa Siam Ceramics, Inc. v. The Secretary
of the Department of Labor and Employment, et al
(BUBAN)
TOPIC: Cancellation of Union Certificate of Registration
FACTS: Petitioner(s): Mariwasa is the employer of the
respondent union.

misrepresentation in violation of Article 239 of the same code.


The Regional Director of DOLE IV-A issued an Order granting
the petition, revoking the registration of respondent, and
delisting it from the roster of active labor unions.
SMMSC-Independent appealed to the Bureau of Labor
Relations. BLR ruled in favor of the respondent, thus, they
remain in the roster of legitimate labor organizations. The
petitioner appealed and insisted that private respondent failed
to comply with the 20% union membership requirement for its
registration as a legitimate labor organization because of the
disaffiliation from the total number of union members of 102
employees who executed affidavits recanting their union
membership Hence, this petition for review on certiorari under
Rule 45 of the Rules of Court.
ISSUES: (1) WON there was failure to comply with the 20%
union membership requirement
(2) WON the withdrawal of 31 union members affected the
petition for certification election insofar as the 30%
requirement is concerned
HELD: (1) Yes (2)

Respondent(s): SMMSC-Independent is the respondent union,


composed of employees of petitioner
Private respondent SMMSC-Independent was issued a
Certificate of Registration as a legitimate labor organization by
the DOLE, Region IV-A. On June 2005, petitioner Mariwasa
Siam Ceramics, Inc. filed a Petition for Cancellation of Union
Registration against private respondent, claiming that the latter
violated Article 234 of the Labor Code for not complying with
the 20% requirement and that it committed massive fraud and

RATIO: (1) On the first issue, while it is true that the


withdrawal of support may be considered as a resignation from
the union, the fact remains that at the time of the unions
application for registration, the affiants were members of
respondent and they comprised more than the required 20%
membership for purposes of registration as a labor union.
Article 234 of the Labor Code merely requires a 20% minimum
membership during the application for union registration. It
does not mandate that a union must maintain the 20%

LABOR 2 | Atty. Aonuevo | BETT DIGEST GROUP |

18

minimum membership
existence.

requirement

all

throughout

its

(2) It appears undisputedly that the 31 union members had


withdrawn their support to the petition before the filing of said
petition. The distinction must be that withdrawals made before
the filing of the petition are presumed voluntary unless there is
convincing proof to the contrary, whereas withdrawals made
after the filing of the petition are deemed involuntary.
Therefore, following jurisprudence, the employees were not
totally free from the employers pressure and so the
voluntariness of the employees execution of the affidavits
becomes suspect. The cancellation of a unions registration
doubtless has an impairing dimension on the right of labor to
self-organization. For fraud and misrepresentation to be
grounds for cancellation of union registration under the Labor
Code, the nature of the fraud and misrepresentation must be
grave and compelling enough to vitiate the consent of a
majority of union members.
CASE 41: The Heritage Hotel Manila v. National Union of
Workers in the Hotel, Restaurant and Allied IndustriesHeritage Hotel Manila Supervisors Chapter (NUWHRAINHHMSC)
Topic: Cancellation of Registration
Facts: Respondent filed with the DOLE a petition for
certification election. The Med-Arbiter ruled in favor of the
certification election and upon appeal by petitioner to the Sec
of Labor, the latter affirmed the ruling of the Med-Arbiter. The
certification election however was not held initially as
scheduled and it was archived. Upon it revival, petitioner
discovered that respondent has not yet submitted its financial
reports as well as the list of its members and it moved to have
the certification election cancelled on that ground. Despite

petitioners opposition, the certification election was held and


respondent was elected as the sole bargaining agent of the
supervisors of petitioner. Petitioner then filed a protest with a
motion to defer certification election results and winner arguing
that once the registration of respondent has already been
cancelled, it would no longer be entitled to be certified as the
exclusive bargaining agent of petitioners supervisors.
Respondent filed its answer claiming that it already complied
with the documentary requirements, albeit delayed. The MedArbiter ruled in favor of respondents and dismissed the
protest, stating that pendency of a petition for cancellation of
registration is not a bar to the holding of a certification election.
Upon appeal to the Sec of Labor, the Sec affirmed the
dismissal of the protest. Aggrieved, petitioner appealed the
decision to the BLR, however, BLR Director Cacdac inhibited
himself as he was the former lawyer of respondent. The Sec of
Labor then took cognizance of the case and again upheld the
certification of election. Petitioner appealed to the CA arguing
that the Sec of Labor should have taken cognizance of the
case, the CA however, ruled in favor respondents and upheld
the Sec of Labors authority to take cognizance of the case.
Petitioner on appeal before the SC argues that once it is
determined that a ground enumerated in Article 239 of the
Labor Code is present, cancellation of registration should
follow; it becomes the ministerial duty of the Regional Director
to cancel the registration of the labor organization, hence, the
use of the word shall. Petitioner points out that the Regional
Director has admitted in its decision that respondent failed to
submit the required documents for a number of years;
therefore, cancellation of its registration should have followed
as a matter of course.
Issue: W/N the certification election was validly done?

LABOR 2 | Atty. Aonuevo | BETT DIGEST GROUP |

19

Held: Yes, the cancellation of the registration would amount to


dissolution of the organization, contrary to RA 9481.
Ratio: It is worth mentioning that the Labor Codes provisions
on cancellation of union registration and on reportorial
requirements have been recently amended by RA 9481, An
Act Strengthening the Workers Constitutional Right to SelfOrganization, Amending for the Purpose Presidential Decree
No. 442, As Amended, Otherwise Known as the Labor Code of
the Philippines. The amendment sought to strengthen the
workers
right
to
self-organization
and
enhance
the Philippines compliance with its international obligations as
embodied in the International Labour Organization (ILO)
Convention No. 87, pertaining to the non-dissolution of
workers organizations by administrative authority. Thus, RA
9481 amended Article 239 to read:
ART. 239. Grounds for Cancellation of Union
Registration.The following may constitute
grounds for cancellation of union registration:
(a) Misrepresentation, false statement or fraud
in connection with the adoption or ratification of
the constitution and by-laws or amendments
thereto, the minutes of ratification, and the list of
members who took part in the ratification;
(b) Misrepresentation, false statements or fraud
in connection with the election of officers,
minutes of the election of officers, and the list of
voters;
(c) Voluntary dissolution by the members.
R.A. No. 9481 also inserted in the Labor Code Article 242-A,
which provides:

ART. 242-A. Reportorial Requirements.The


following are documents required to be
submitted to the Bureau by the legitimate labor
organization concerned:
(a) Its constitution and by-laws, or amendments
thereto, the minutes of ratification, and the list of
members who took part in the ratification of the
constitution and by-laws within thirty (30) days
from adoption or ratification of the constitution
and by-laws or amendments thereto;
(b) Its list of officers, minutes of the election of
officers, and list of voters within thirty (30) days
from election;
(c) Its annual financial report within thirty
(30) days after the close of every fiscal year;
and
(d) Its list of members at least once a year or
whenever required by the Bureau.
Failure to comply with the above requirements
shall not be a ground for cancellation of union
registration but shall subject the erring officers
or members to suspension, expulsion from
membership, or any appropriate penalty.
ILO Convention No. 87, which we have ratified in 1953,
provides that workers and employers organizations shall not
be liable to be dissolved or suspended by administrative
authority. The ILO has expressed the opinion that the
cancellation of union registration by the registrar of labor
unions, which in our case is the BLR, is tantamount to

LABOR 2 | Atty. Aonuevo | BETT DIGEST GROUP |

20

dissolution of the organization by administrative authority when


such measure would give rise to the loss of legal personality of
the union or loss of advantages necessary for it to carry out its
activities, which is true in our jurisdiction. Although the ILO has
allowed such measure to be taken, provided that judicial
safeguards are in place, i.e., the right to appeal to a judicial
body, it has nonetheless reminded its members that dissolution
of a union, and cancellation of registration for that matter,
involve serious consequences for occupational representation.
It has, therefore, deemed it preferable if such actions were to
be taken only as a last resort and after exhausting other
possibilities with less serious effects on the organization.
CASE 42: Takata (Philippines) Corporation v. BLR, G.R.
No. 196276, June 4, 2014 TORRES
REFER TO CASE 34
CASE 43: Faculty Union v. Bitonio, G.R. No. 131235,
November 16, 1999 TUTAAN
TOPIC: Union- Members Relation
Facts: Private respondent Marinio et al were duly elected
officers of UST faculty (USTFU). The union has a 5-year CBA
with its employer and is set to expire on May 31, 1998. The
Secretary General of USTFU posted a notice addressed to all
USTFU members announcing a general assembly to be held
on 05 October 1996 and to elect USTFUs next set of officers.
Through the notice, the members were also informed of the
constitution of a Committee on Elections (COMELEC) to
oversee the elections. On 01 October 1996, some of herein
appellants filed a separate petition with the Med-Arbiter,
DOLE-NCR, directed against herein appellees and the
members of the COMELEC. The petition alleged that the
COMELEC was not constituted in accordance with USTFUs
constitution and by-laws (CBL) and that no rules had been

issued to govern the conduct of the 05 October 1996 election.


On 04 October 1996, the med-arbiter issued a temporary
restraining order against herein appellees enjoining them from
conducting the election scheduled on 05 October 1996.
On 04 October 1996, and as earlier announced by the UST
secretary general, a general faculty assembly as earlier
requested by various UST faculty club president was held
scheduled. The general assembly was attended by members
of the USTFU and, as admitted by the appellants, also by 'nonUSTFU members [who] are members in good standing of the
UST Academic Community Collective Bargaining Unit'
On
this occasion, appellants were elected as USTFUs new set of
officers by acclamation and clapping of hands. The election of
the appellants came about upon a motion of one Atty. Lopez,
admittedly not a member of USTFU, that the USTFU CBL and
'the rules of the election be suspended and that the election be
held on the same occasion.
On 03 December 1996, appellants and UST allegedly entered
into another CBA covering the period from 01 June 1996 to 31
May 2001. Appellees reiterated their earlier stand that
appellants were usurping the formers duties and functions and
should be stopped from continuing such acts. Med-Arbiter
declared the election violative of the CBL while BLR
director Bitonio upheld the decision with a ruling that the CBL
which constituted the covenant between the union and its
members could not be suspended during the general
assembly of all faculty members, since it had not been
convened not been authorized by the union.
As ruled by BLR Director Bitonio: To accept appellants' claim
to legitimacy on the foregoing grounds is to invest in
appellants the position, duties, responsibilities, rights and

LABOR 2 | Atty. Aonuevo | BETT DIGEST GROUP |

21

privileges of USTFU officers without the benefit of a lawful


electoral exercise as defined in USTFU's CBL and Article
241(c) of the Labor Code. Not to mention the fact that labor
laws prohibit the employer from interfering with the employees
in the latter' exercise of their right to self-organization. To
allow appellants to become USTFU officers on the strength of
management's recognition of them is to concede to the
employer the power of determining who should be USTFU's
leaders. This is a clear case of interference in the exercise by
USTFU members of their right to self-organization.
Issues: (1) WON the Collective Bargaining Unit of all the
faculty members in that General Faculty Assembly had the
right in that General Faculty Assembly to suspend the
provisions of the Constitution and By-Laws of the USTFU
regarding the elections of officers of the union. NO.
(2) WON the suspension of the provisions of the Constitution
and By-Laws of the USTFU in that General Faculty Assembly
is valid pursuant to the constitutional right of the Collective
Bargaining Unit to engage in peaceful concerted activities for
the purpose of ousting the corrupt regime of the private
respondents NO.
(3) WON the overwhelming ratification of the Collective
Bargaining Agreement executed by the petitioners in behalf of
the USTFU with the University of Santo Tomas has rendered
moot and academic the issue as to the validity of the
suspension of the Constitution and By-Laws and the elections
of October 4, 1996 in the General Faculty Assembly. NO.
Ratio: (1). No, the October 4, 1996 election cannot properly
be called a union election, because the procedure laid down in
the USTFUs CBL for the election of officers was not followed.

It could not have been a certification election either, because


representation was not the issue, and the proper procedure for
such election was not followed. The participation of non-union
members in the election aggravated its irregularity. A union
election is held pursuant to the unions constitution and
bylaws, and the right to vote in it is enjoyed only by union
members. A union election should be distinguished from
a certification election, which is the process of
determining, through secret ballot, the sole and exclusive
bargaining agent of the employees in the appropriate
bargaining unit, for purposes of collective bargaining.
Specifically, the purpose of a certification election is to
ascertain whether or not a majority of the employees wish
to be represented by a labor organization and, in the
affirmative case, by which particular labor organization.
In a certification election, all employees belonging to the
appropriate bargaining unit can vote. Therefore, a union
member who likewise belongs to the appropriate bargaining
unit is entitled to vote in said election. However, the reverse is
not always true; an employee belonging to the appropriate
bargaining unit but who is not a member of the union cannot
vote in the union election, unless otherwise authorized by the
constitution and bylaws of the union. Verily, union affairs and
elections cannot be decided in a non-union activity.
Self-organization is a fundamental right guaranteed by the
Constitution and the Labor Code. Corollary to this right is the
prerogative not to join, affiliate with or assist a labor union.
Therefore, to become a union member, an employee must not
only signify the intent to become one, but also take some
positive steps to realize that intent. The procedure for union
membership is usually embodied in the unions CBL. An
employee who becomes a union member acquires the rights

LABOR 2 | Atty. Aonuevo | BETT DIGEST GROUP |

22

and he concomitant obligations that go with the new status


and becomes bound by the unions rules and regulations.
(2) No. (Note: Petitioners contend that the October 4, 1996
assembly suspended the unions CBL. They aver that the
suspension and the election that followed were in accordance
with their constituent and residual powers as members of the
collective bargaining unit to choose their representatives for
purposes of collective bargaining. Again they cite the
numerous anomalies allegedly committed by the private
respondents as USTFU officers.)
As ruled by the SC, a union CBL is a covenant between the
union and its members and among members. Where ILO
Convention No. 87 speaks of a unions full freedom to draw up
its constitution and rules, it includes freedom from interference
by persons who are not members of the union.
The
democratic principle that governance is a matter for the
governed to decide upon applies to the labor movement which,
by law and constitutional mandate, must be assiduously
insulated against intrusions coming from both the employer
and complete strangers if the 'protection to labor clause' of the
constitution is to be guaranteed. By appellants own evidence,
the general faculty assembly of 04 October 1996 was not a
meeting of USTFU. It was attended by members and nonmembers alike, and therefore was not a forum appropriate for
transacting union matters. The person who moved for the
suspension of USTFUs CBL was not a member of USTFU.
Allowing a non-union member to initiate the suspension of a
unions CBL, and non-union members to participate in a union
election on the premise that the unions CBL had been
suspended in the meantime, is incompatible with the freedom
of association and protection of the right to organize. If a

member of the union dislikes the provisions of the by-laws he


may seek to have them amended or may withdraw from the
union; otherwise he must abide by them.
Under Article XVII of USTFUs CBL, there is also a specific
provision for constitutional amendments.
What is clear
therefore is that USTFUs CBL provides for orderly procedures
and remedies which appellants could have easily availed
[themselves] of instead of resorting to an exercise of their socalled residual power'.
(3) No, the ratification of the new CBA executed between the
petitioners and the University of Santo Tomas management did
not validate the void October 4, 1996 election. Ratified were
the terms of the new CBA, not the issue of union leadership -a matter that should be decided only by union members in the
proper forum at the proper time and after observance of proper
procedures.
CASE 44: Salunga v. CIR, 21 SCRA 216 (1967); BUBAN
TOPIC: Admission and Discipline of Members
FACTS: San Miguel Brewery (SMB) entered into a CBA with
the National Brewery andAllied Industries Labor Union of the
Philippines (NBAILUP-PAFLU, otherwiseknown as the Union).
Section 3 of the CBA (a Closed Shop Agreement)3

The company agrees to require as a condition of employment of those workers


covered by this agreement who either are members of the UNION on the date of the
signing of this agreement, or mayjoin the UNION during the effectivity of this
agreement, that they shall not voluntarily resign from the UNION earlier than thirty (30)
days before the expiry date of this agreement as provided in ArticleXIII hereof,
provided, however, that nothing herein contained shall be construed to require
thecompany to enforce any sanction whatsoever against any employee or worker who
fails to retain hismembership in the UNION as hereinbefore stated, for any cause
other than voluntary resignation ornon-payment of regular union dues on the part of
said employee or worker.

LABOR 2 | Atty. Aonuevo | BETT DIGEST GROUP |

23

Petitioner Francisco Salunga was a member of the Union


since 1953. Due to afalling out with the Union, he tendered his
resignation from the Union, whichaccepted it and transmitted it
to the Company with a request for the immediate
implementation of said section 3. SMB informed Salunga that
his aforementioned resignation would result in the termination
of his employment, and in view of saidsection, Salunga wrote
a letter to the Union withdrawing or revoking his resignation
and advising the Union to continue deducting his monthly
union dues. He, then, furnished a copy of this communication
to SMB. The latter, in turn,notified the Union of the receipt of
said copy and that "in view thereof, we shall not take any
action on this case and shall consider Mr. Francisco Salunga
still a member of your union and continue deducting his union
dues.
However, the Union told SMB that Salungas membership
could not be reinstated and insisted on his separation from the
service, conformably with the stipulation above-quoted. SMB
sent a reply clarifying theinstructions of the Union to terminate
the employment of Salunga. The Union reiterated its request
for implementation of said section 3, for which reason, SMB
notified Salunga that, in view of said letter and the
aforementioned section, theyhave to terminate his
employment, although with regret.
Meanwhile, Salunga sought the intervention of PAFLU's
National President, Cipriano Cid, to which the Union was
affiliated, for a review of the latter's action. PAFLU gave due
course to petitioner's request for review.
Cid advised Salunga that PAFLU had found no ground to
review the action taken by the Union and that, on the
expiration of the 15-day grace granted to him by the Company,
the decision thereof to terminate his services would take effect.

Salunga then notified the PAFLU that he was appealing to its


supreme authority the PAFLU National Convention and
requested that action on his case bedeferred until such time as
the Convention shall have acted on his appeal.Furthermore,
he asked SMB to maintain the status quo, in the meantime.
This notwithstanding, at the close of the business hours, on
October 15, 1961, Salunga was discharged from the
employment of the Company, through its assistant-secretary
and vice-president, herein respondent Miguel Noel.
Hence, the complaint in the CIR. CIR decided in favor of
Salunga. Defendants (including SMB) all guilty of ULP.
Ordered Union to readmit petitioner as member, and Company
to reinstate him with backwages. On Motion for
Reconsideration, CIR reversed its earlier decision. Hence, this
appeal by the petitioner.
ISSUES: (1) WON the Union is guilty of ULP
(2) WON SMB (the employer) is guilty of ULP.
HELD: (1) Yes
(2) No
RATIO: (1) Although Salunga had resigned from the Union and
the latter had accepted the resignation, Salunga had, soon
later upon learning that his withdrawal from the Union would
result in his separation from the Company, owing to the
closed-shop provision above referred to revoked or
withdrawn said resignation, and the Union refused to consent
thereto without any just cause therefor. The Union had not only
acted arbitrarily in not allowing petitioner to continue his
membership but the trial Judge also found said refusal of the
Union officers to be due to his critical attitude towards certain
measures taken or sanctioned by them. The record is clear
that, feeling dejected by the inaction of the union officials onhis

LABOR 2 | Atty. Aonuevo | BETT DIGEST GROUP |

24

grievances and objections to what he believed were illegal


disbursements ofunion funds, coupled with the fact that he
was later removed from his position as a union steward
without his knowledge, as well as the fact that the union did
not honor the power of attorney executed in his favor by
Alejandro Miranda, a co-worker, for the collection of Miranda's
indebtedness of P60.00 to him, he submitted his letter of
resignation from the union. The resolution appealed from
cannot be affirmed without, in effect, nullifying the right which,
independently of the constitution and by-laws of the Union, is
part and parcel of the freedom of speech guaranteed in the
Constitution of our Republic, as a condition sine quanon to the
sound growth and development of labor organizations and
democratic institutions.
(2) SMB did not engage in ULP. It was shown that SMB itself
was even reluctant if not unwilling to discharge the
petitioner.
When the Union first informed SMB of Salungas resignation
and urged implementation of section 3 of the bargaining
contract, SMB advised Salunga ofthe provision, thereby
intimating that he had to withdraw his resignation in order to
keep his employment. Besides, SMB notified the Union that it
(SMB) would not take any action on the case and would
consider Salunga "still a member" ofthe Union.
When the latter, thereafter, insisted on petitioner's discharge,
SMB stilldemurred and explained it was not taking sides and
that its stand was promptedmerely by "humane"
considerations, springing from the belief that petitioner
hadresigned from the Union without realizing its effect upon his
employment. And, as the Union reiterated its demand, SMB
notified petitioner that it had no otheralternative but to

terminate his employment, and dismissed him from the


service,although with "regret". Under these circumstances,
SMB was not "unfair" to the petitioner.
On the contrary, it did not merely show a commendable
understanding of and sympathy for his plight. It even tried to
help him, although to such extent only as was consistent with
its obligation to refrain from interfering in purely internal
affairsof the Union. At the same time, SMB could not safely
inquire into the motives ofthe Union officers, in refusing to
allow Salunga to withdraw his resignation.
Inasmuch as the true motives were not manifest, without such
inquiry, and Salunga had concededly tendered his resignation
of his own free will, the arbitrary nature of the decision of said
officers was not such as to be apparent and to justify the
company in regarding said decision unreasonable. Upon the
other hand, SMB cannot be blamed for assuming the contrary,
for Salunga had appealed to the National Officers of the
PAFLU and the latter had sustained the Union. SMB was
therefore justified in presuming that the PAFLU had inquired
into all relevant circumstances, including the motives of the
Union Officers.
CASE 45: v. Inciong, 121 SCRA 444 (1983) ESTOMO
Topic: Admission and Discipline of Members
Facts: Petitioners are members of Amigo Employees UnionPAFLU (AEU-PAFLU), the then recognized sole bargaining
representative of the employees of Amigo Manufacturing Corp.
(Amigo). AEU-PAFLU and Amigo had an existing CBA with a
closed-shop provision. Prior to the expiration of the CBA, Villar
et al, for and on behalf of the newly formed union, the
Federation of Union of Rizal, filed for a certification of election

LABOR 2 | Atty. Aonuevo | BETT DIGEST GROUP |

25

with the Med-Arbiter. AEU-PAFLU opposed the petition arguing


ethical reasons based on the by-laws of AEU-PAFLU. The
Med-Arbiter dismissed the certification election. In light of the
filing for certificate election by Villar et al, AEU-PAFLU created
a trial committee specifically to address the actions of Villar et
al. AEU-PAFLU then sent a letter to the former, informing them
that their actions bismerched the reputation of AEU-PAFLU
and thus would result to their expulsion. Coupled with the
closed-shop provision in the CBA, Amigo then sent a letter to
Villar et al that they would be dismissed from service due to
their expulsion from AEU-PAFLU. Villar et al averred that they
did not intended to bismerch the reputation of AEU-PAFLU, but
rather, they were simply exercising their right to selforganization. Villar et al then filed a complaint for unfair labor
practices against private respondents with the Med-Arbiter and
the Arbiter ruled in favor of private respondets. Upon appeal to
the Sec of Justice, the Sec of Justice affirmed the decision of
the Med-Arbiter ruling that the action of Villar et al was
contrary to the by-laws of AEU-PAFLU, further ruling that at the
time they filed their petition for certification electon, they were
still members of the above-mentioned union. Furthermore, the
CBA provision between Amigo and AEU-PAFLU was still in
force and effect.
Issue: W/N the dismissal of petitioners was proper?
Held: Yes, they were still members of AEU-PAFLU at the time
they filed for the certification election and their newly formed
organization has not yet attained the requisite personality.
Ratio: The dismissal of petitioners is not a limitation to the
right of assembly or association, which may be exercised with
or without said registration. The latter is merely a condition
sine qua non for the acquisition of legal personality by labor
organizations, associations or unions and the possession of

the 'rights and privileges granted by law to legitimate labor


organizations.' The Constitution does not guarantee these
rights and privileges, much less said personality, which are
mere statutory creations, for the possession and exercise of
which registration is required to protect both labor and the
public against abuses, fraud, or impostors who pose as
organizers, although not truly accredited agents of the union
they purport to represent. Such requirement is a valid exercise
of the police power, because the activities in which labor
organizations, associations and union or workers are engaged
affect public interest, which should be protected.
Simply put, the Amigo Employees Union (Independent) Which
petitioners claim to represent, not being a legitimate labor
organization, may not validly present representation issues.
Therefore, the act of petitioners cannot be considered a
legitimate exercise of their right to self-organization. Hence,
We affirm and reiterate the rationale explained in Phil
Association of Free Labor Unions vs. Sec. of Labor case,
supra, in order to protect legitimate labor and at the same time
maintain discipline and responsibility within its ranks.
CASE 46: Bugay v. Kapisanan ng mga Manggagawa sa
MRR, 4 SCRA 487 (1962) TORRES
FACTS: Paulino Bugay filed against Kapisana ng Mga
Manggagawa Sa Manila Railroad Company before the CFI of
Manila an action for moral damages arising out of unfair labor
practice committed by said union which subject of a decision
rendered by the CIR and affirmed by the SC finding the said
union guilty as charged. The action for moral damages was
based under RA 875, as found by the CIR and the SC, and
that the plaintiff suffered moral damages for mental anguish,
anxiety, social humiliation and besmirched reputation among

LABOR 2 | Atty. Aonuevo | BETT DIGEST GROUP |

26

the thousand
P20,000.00.

employees

of

MRR

Co.

amounting

to

HELD: YES. The union was in violation of justice and fair play
in the expulsion of Bugay.

Defendant filed a motion to dismiss on the ground that the


complaint does not state facts sufficient to constitute a cause
of action in that neither the decision of the Court of Industrial
Relations nor that of the Supreme Court contain any statement
that the unfair labor practice act imputed to the defendant was
false or fabricated as in fact the decision of the Supreme Court
ordering plaintiff's reinstatement was merely based on "the
regularity and validity of the proceedings and the means
adopted by the union and its officers in effecting (his)
expulsion." This contention having been sustained, the lower
court dismissed the complaint with costs against plaintiff

RATIO: The committee of three board members assigned to


summon Bugay failed to serve notice upon him because he
was then in Lucena, Quezon. Why all these proceedings were
continued by the respondents inspite of Bugay's absence
remains unexplained in the record. But one thing is certain,
whatever might be the merits of the charge filed by respondent
Olazo against him, Bugay did not have sufficient opportunity to
defend himself. Such proceedings, being violative of the
elementary rule of justice and fair play, can not give validity to
any act done pursuant thereto.

It appears that Bugay was formerly an auditor of the defendant


union and at the same time, the payroll clerk of MRR Co. He
was requested by the secretary-treasurer of MRR to deliver
certain documents of the union which were in his possession.
In compliance, he delivered them without consulting the
officers of the union. The documents were then used by MRR
to charge the president of the union, Vicente Olazo, for
falsification of commercial documents.
Although the city fiscal dismissed the charge, Bugay was still
charged by the union for disloyalty and conduct unbecoming of
a union member, and the appellant was expelled from the
union.
ISSUE: WON such proceedings in the expulsion Bugay from
the union were in violation of the elementary rule of justice and
fair play.

An examination of the chapters to the Kapisanan board of


directors (Exhs. 3 6 to 28) s hows tha t all of the votes, those
of the Hondagua Chapters and Engineering Manila Yard
Chapter (Exhs. 14 and 17) were not validly cast. Under the
Kapisanan's constitution and by-laws, relied upon by the
parties, before a resolution of general application may be
enforced, and a resolution terminating union membership is
one, it must receive the sanction of majority of the chapters
within ten (10) days. The resolution had to be affirmed by the
chapters. Only the two abovenamed chapters, however, acted
on the resolution within the prescribed period. Even under the
assumption that the proceedings against Bugay were not
irregular, the resolution in question never had any valid effect
on his union membership. His affiliation with the Kapisanan
was never terminated. That being the case, Bugay is entitled
to all the rights and obligations appertaining to every member
of the Kapisanan.
The result was that because of his expulsion he was subjected
to humiliation and mental anguish with the consequent lose of

LABOR 2 | Atty. Aonuevo | BETT DIGEST GROUP |

27

his good name and reputation. This is especially so


considering that the members of the union from which he was
expelled amounted to around 20,000 more or less, It is,
therefore, an error for the lower court to hold that the complaint
does not state sufficient cause of action for the relief claimed
by appellant.
With regard to the contention that this claim o f or mo
damages should have been included by appellant in his
charge for unfair labor practice filed against the union with the
Court of Industrial Relations, suffice it to state that the same
does not come within the jurisdiction of that court. This is a
matter that has to be looked into by the regular courts.
CASE 47: Tancinco v. Calleja, 157 SCRA 203 (1988)
TUTAAN
TOPIC: Election of Officers (Who may vote)
FACTS: Private respondents are the prime organizers of
Imperial Textile Mills Inc. Monthly Employees Association (ITMMEA). While said respondents were preparing to file a petition
for direct certification of the Union as the sole and exclusive
bargaining agent of ITM's bargaining unit, the union's VicePresident, Carlos Dalmacio was promoted to the position of
Department Head, thereby disqualifying him for union
membership. Said incident, among others led to a strike
spearheaded by Lacanilao group, respondents herein. Another
group however, led by herein petitioners staged a strike inside
the company premises. After four (4) days the strike was
settled. On May 10, 1986 an agreement was entered into by
the representatives of the management, Lacanilao group and
the Tancinco group the relevant terms of which are as follows:
1. That all monthly-paid employees shall be United under one
union, the ITM Monthly Employees Association (ITM-MEA), to
be affiliated with ANGLO;

2. That the management of ITM recognizes ANGLO as the


sole and exclusive bargaining agent of all the monthly-paid
employees;
3. That an election of union officers shall be held on 26 May
l986, from 8:00 a.m. to 5:00 p.m.;
4. That the last day of filing of candidacy shall be on l9 May
l986 at 4:00 p.m.;
5. That a final pre-election conference to finalize the list of
qualified voters shall be held on 19 May 1986, at 5:00 p.m.
On May 19, 1986, a pre-election conference was held, but the
parties failed to agree on the list of voters. During the preelection conference attended by MOLE officers, ANGLO
through its National Secretary made a unilateral ruling
excluding some 56 employees consisting of the:Manila office
employees;
a. members of Iglesia ni Kristo;
b. non-time card employees;
c. drivers of Mrs. Salazar; and
d. the cooperative employees of Mrs. Salazar.
On May 26, 1986, the election of officers was conducted under
the supervision of MOLE wherein the 56 employees in
question participated but whose votes were segregated
without being counted. Lacanilao's group won. Lacanilao
garnered 119 votes with a margin of three (3) votes over
Tancinco prompting petitioners to make a protest. Thereafter,
petitioners filed a formal protest with the Ministry of Labor
Regional Office in San Fernando, Pampanga claiming that the
determination of the qualification of the 56 votes is beyond the
competence of ANGLO. Private respondents maintain the
contrary on the premise that definition of union's membership
is solely within their jurisdiction. MOLE's Med Arbiter issued an
order directing the opening and counting of the segregated

LABOR 2 | Atty. Aonuevo | BETT DIGEST GROUP |

28

votes. From the said order private respondents appealed to


the Bureau of Labor Relations (BLR) justifying the
disenfranchisement of the 56 votes. Private respondents
categorized the challenged voters into four groups namely: (a)
the Manila Employees, (b) that they are personal employees of
Mr. Lee; (c) The Iglesia ni Kristo, that allowing them to vote
will be anomalous since it is their policy not to participate in
any form of union activities; (d) the non-time card employees,
that they are managerial employees; and (e) the employees of
the cooperative as non-ITM employees.
BLR rendered a decision holding the exclusion of the 56
employees as arbitrary, whimsical, and wanting in legal basis
but set aside the challenged order of July 26, 1986 on the
ground that 51 ** of 56 challenged voters were not yet union
members at the time of the election per April 24, 1986 list
submitted before the Bureau on the ground that their names
do not appear in the records of the Union submitted to the
Labor Organization Division of the Bureau of Labor on April 24,
1986. The decision directed among others the proclamation of
Lacanilao's group as the duly elected officers and for ITM-MEA
to absorb in the bargaining unit the challenged voters unless
proven to be managerial employees. Petitioners' motion for
reconsideration was likewise denied. Hence, this petition.
Issue: WON the BLR is correct in ruling that the petitioners
cannot be allowed to vote because they are not yet members
of the union at the time of conduct of election of officers? NO
Ratio: The BLR erred. BLR Decision reversed; petition
granted. Submission of the employees names with the BLR as
qualified members of the union is not a condition sine qua
non to enable said members to vote in the election of union's
officers. It finds no support in fact and in law. Per public
respondent's findings, the April 24, 1986 list consists of 158

union members only wherein 51 of the 56 challenged voters'


names do not appear. Adopting however a rough estimate of a
total number of union members who cast their votes of some
333 and excluding therefrom the 56 challenged votes, if the list
is to be the basis as to who the union members are then public
respondent should have also disqualified some 175 of the 333
voters. It is true that under article 242(c) of the Labor Code, as
amended, only members of the union can participate in the
election of union officers. The question however of eligibility to
vote may be determined through the use of the applicable
payroll period and employee's status during the applicable
payroll period. The payroll of the month next preceding the
labor dispute in case of regular employees and the payroll
period at or near the peak of operations in case of employees
in seasonal industries. (Refer to the dates mentioned above:
April and May. Thats why the preceding month before the
labor dispute)
In the case before Us, considering that none of the parties
insisted on the use of the payroll period-list as voting list and
considering further that the 51 remaining employees were
correctly ruled to be qualified for membership, their act of
joining the election by casting their votes on May 26, 1986
after the May 10, 1986 agreement is a clear manifestation of
their intention to join the union. They must therefore be
considered ipso facto members thereof Said employees
having exercised their right to unionism by joining ITM-MEA
their decision is paramount.
Their names could not have been included in the list of
employee submitted on April 24, 1986 to the Bureau of Labor
for the agreement to join the union was entered into only on
May 10, 1986. Indeed the election was supervised by the
Department of Labor where said 56 members were allowed to

LABOR 2 | Atty. Aonuevo | BETT DIGEST GROUP |

29

vote. Private respondents never challenged their right to vote


then.
Existence of a CBA and cordial relationship developed
between the union and the management should not be a
justification to frustrate the decision of the union members as
to who should properly represent them in the bargaining unit.
Neither may the inclusion and counting of the 56 segregated
votes serve to disturb the existing relationship with
management as feared by herein private respondents.
Respondents themselves pointed out that petitioners joined
the negotiating panel in the recently concluded CBA. This fact
alone is conclusive against herein petitioners and hence will
estop them later if ever, from questioning the CBA which
petitioners concurred with. Furthermore, the inclusion and
counting of the 56 segregated votes would not necessarily
mean success in favor of herein petitioners as feared by
private respondents herein.
CASE 48: Kapisanan v. Trajano, 134 SCRA 236 (1985)
(BUBAN)
TOPIC: Election of Election
FACTS: A written request for accounts examination of the
financial status of the Kapisanan ng Manggagawang
Pinagyakap (KMP) Labor Union (Union for brevity), the
existing labor union at Franklin Baker Company in San Pablo
City, was filed by private respondent Catalino Silvestre and
thirteen (13) other employees, who are also members of the
said Union. Acting on said request, Union Account Examiner
Florencio R. Vicedo of the Ministry of Labor and Employment
conducted the necessary investigation and, thereafter,
submitted a report on disallowed expenditures.

Based on the foregoing revelations, private respondents filed a


petition for the expulsion of the union officers on the ground
that they committed gross violation of the Labor Code and, the
constitution and by-laws of the Union, particularly the
provisions of Sections 6 and 7 thereof.
In their Answer, the union officers denied the imputation and
argued that the disallowed expenditures were made in good
faith; that the same conduced to the benefit of the members;
and, that they are willing to reimburse the same from their own
personal funds. They likewise asserted that they should not be
held accountable for the non-production of the books of
accounts of the Union for the years 1977, 1978 and 1979
because they were not the officers then and not one of the
former officers of the Union had turned over to them the
records in question. Further, they averred that the nonratification of the constitution and by-laws of the Union and the
non-segregation of the Union funds occurred before they
became officers and that they have already been correcting
the same.
Med-Arbiter ordered the holding of a referendum, to be
conducted under the supervision of the Bureau of Labor
Relations, to decide on the issue of whether to expel or
suspend the union officers from their respective positions.
ISSUE: WON union officer may be held accountable for
liability incurred prior to their assumption as officer
HELD: NO
RATIO: The alleged falsification and misrepresentation of
herein union officers were not supported by substantial

LABOR 2 | Atty. Aonuevo | BETT DIGEST GROUP |

30

evidence. The fact that they disbursed the amount of


P1,278.00 from Union funds and later on was disallowed for
failure to attach supporting papers thereon did not of itself
constitute falsification and/or misrepresentation. The
expenditures appeared to have been made in good faith and
the amount spent for the purpose mentioned in the report, if
concurred in or accepted by the members, are reasonable;
and the repudiation of both private respondents to the highly
sensitive position of auditor at the election, is a convincing
manifestation and demonstration of the union membership's
faith in the herein officers' leadership on one hand and a clear
condonation of an act they had allegedly committed.
The Court should never remove a public officer for acts done
prior to his present term of office. To do otherwise would be to
deprive the people of their right to elect their officers. When the
people have elected a man to office, it must be assumed that
they did this with knowledge of his life and character, and that
they disregarded or forgave Ms faults or misconduct, if he had
been guilty of any. It is not for the court, by reason of such
faults or misconduct to practically overrule the will of the
people.
CASE 49: Gabriel v. Secretary of Labor, G.R. No. 115949,
March 16, 2000 ESTOMO
Topic: Payment of Attorneys Fees and Special Assessments
Facts: Petitioners comprises the Executive Board of
SolidBank Union, the duly recognized collective bargaining
agent of SolidBank and private respondents are the rank-andfile employees who are also members of the said Union. The
EB decided to retain anew the service of Atty. Ignacio P.
Lacsina (now deceased) as union counsel in connection with
the negotiations for a new CBA. The Board called a general

membership meeting for the purpose. At the said meeting, the


majority of all union members approved and signed a
resolution confirming the decision of the executive board to
engage the services of Atty. Lacsina as union counsel. As
approved, the resolution provided that 10% of the total
economic benefits that may be secured through the
negotiations be given to Atty. Lacsina as attorneys fees. It also
contained an authorization for SolidBank Corporation to checkoff said attorneys fees from the first lump sum payment of
benefits to the employees under the new CBA and to turn over
said amount to Atty. Lacsina and/or his duly authorized
representative. After the signing of the new CBA, SolidBank
then, on request of the Union, made payroll deductions for
attorneys fees from the CBA benefits paid to the union
members in accordance with the abovementioned resolution.
Private respondents then filed a complaint for illegal deduction
of attorneys fees with the Med-Arbiter. The Med-Arbiter ruled
in favor of the private respondents and ordered petitioner to
refund the amount deducted from the employees. Upon appeal
to the Sec of Labor, the Sec of Labor ruled that the employees
should be made to shoulder the expenses of attorneys fees
through their Union. Upon appeal to this Court, petitioner
maintains that General Membership Resolution authorizing the
bank to check-off attorneys fee from the first lump sum
payment of the benefits to the employees under the new CBA
satisfies the legal requirements for such assessment. Private
respondents, on the other hand, claim that the check-off
provision in question is illegal because it was never submitted
for approval at a general membership meeting called for the
purpose and that it failed to meet the formalities mandated by
the Labor Code.
Issue: W/N the Sec of Labor erred in ruling that the attorneys
fees should be paid through the employees Union?

LABOR 2 | Atty. Aonuevo | BETT DIGEST GROUP |

31

Held: No, attorneys fees may not be deducted or checked off


from an individuals salary without his consent.
Ratio: The pertinent legal provisions on check-offs are found
in Article 222 (b) and Article 241 (o) of the Labor Code.

fees and representation expenses. These are: 1) authorization


by a written resolution of the majority of all the members at the
general membership meeting called for the purpose; (2)
secretarys record of the minutes of the meeting; and (3)
individual written authorization for check off duly signed by the
employees concerned.

Article 222 (b) states:


"No attorneys fees, negotiation fees or similar
charges of any kind arising from any collective
bargaining negotiations or conclusions of the
collective agreement shall be imposed on any
individual
member
of
the
contracting
union: Provided, however, that attorneys fees
may be charged against union funds in an
amount to be agreed upon by the parties. Any
contract, agreement or arrangement of any sort
to the contrary shall be null and void."

Clearly, attorneys fees may not be deducted or checked off


from any amount due to an employee without his written
consent.
After a thorough review of the records, we find that the
General Membership Resolution of October 19, 1991 of the
SolidBank Union did not satisfy the requirements laid down by
law and jurisprudence for the validity of the ten percent (10%)
special assessment for unions incidental expenses, attorneys
fees and representation expenses. There were no individual
written check off authorizations by the employees concerned
and so their employer cannot legally deduct the assessment.

Article 241 (o) provides:


"Other than for mandatory activities under the
Code, no special assessment, attorneys fees,
negotiation fees or any other extraordinary fees
may be checked off from any amount due to an
employee without
an
individual
written
authorization duly signed by the employee. The
authorization should specifically state the
amount, purpose and beneficiary of the
deduction."
Article 241 has three (3) requisites for the validity of the
special assessment for unions incidental expenses, attorneys

CASE 50: Galvadores v. Trajano, 144 SCRA 138 (1986)


TORRES
TOPIC: Government Regulations; Union-Member Relations;
Mandatory Activity
FACTS: Petitioner employees of the PLDT and members of
respondent Free Telephone Workers Union, now the
Manggagawa ng Komunikasyon sa Pilipinas (Union), question
the legality of the check-off for attorneys fees amounting to
P1M of respondent Atty. Jose C. Espinas (Respondent
Counsel) from the monetary benefits awarded to PLDT
employees in a deadlocked collective bargaining agreement
negotiations between the PLDT and the Union.

LABOR 2 | Atty. Aonuevo | BETT DIGEST GROUP |

32

Respondent counsel has been the legal counsel of respondent


Union since 1964, but he was hired on a case to case
contingent basis. The Union once again requested his
appearance as counsel in the ongoing labor dispute with the
PLDT. In consideration, the Union bound itself to compensate
him for fees and expenses on a contingent basis, and the
amount shall be 10% of any improvement, with retroactive
effect, of PLDTs last offer to the deadlock in CBA negotiations.
The Minister of Labor awarded the across-the-board wage
increases of P330/month effective November 9, 1982;
P155/month effective November 9, 1983, and P155/month
effective November 9, 1984, in addition to the Christmas
bonus of 1 month pay per employee effective December,
1983, and other fringe benefits. As will be noted, there were
improvements obtained from PLDTs last offer.
The Executive Board of the Union passed a resolution
requesting PLDT to deduct P115.00 per employee for the legal
services extended to the Union by Respondent Counsel.
The petitioners numbering 5,258, filed a letter-complaint
before the MOLE through petitioner Carlos Galvadores,
assailing the imposition of P130.00 (later corrected to
P155.00) per employee as attorneys fees of respondent
counsel. Annexed to the complaint were the written statements
of the employee authorizing Galvadores to act for and in their
behalf. Petitioners took the position that the attorneys fees of
respondent counsel were not only unreasonable but also in
violation of Article 242(o) of the Labor Code. The deductions
cannot be given legal effect by a mere Board resolution but
needs the ratification by the general membership of the Union.

The Union filed a Manifestation that about 6,067 members of


the Union ratified the October 29, 1983 resolution of the
legislative council in a plebiscite called for that purpose.
Petitioners questioned the plebiscite on the ground that
Question No. 2, which reads:
Question No. 2. Do you approve of the use of P1
million (P500,000.00 to be withdrawn from PECCI
and another P500,000.00 from IBAA) from our CBA
negotiation fund together with the attorneys fees
(P1 million) that was collected and to be loaned to
the MKP/FTWU, as our counterpart of the seed
money to start the housing program as agreed by
the PLDT management and our union panel and
included in the award of the MOLE?
was misleading and deceptive as it assumed that there was no
dispute regarding the deduction of attorneys fees from the
monetary benefits awarded to PLDT employees.
Respondent Director of the Bureau of Labor Relations
dismissed petitioners complaint for lack of merit reasoning that
the outcome of the plebiscite negates any further question on
the right of the union counsel to collect the amount of P115
from each of the employees involved. The individual written
authorization of all the employees must first be obtained
before any assessment can be made against the monetary
benefits awarded to them pursuant to Article 242(o) of the
Labor Code. Assuming that Respondent Counsel is entitled to
attorneys fees, the same should be taken from Union funds.
In his Comment, the Solicitor General agrees with petitioners
that the issue presented is squarely covered by Article 222(b)
of the Labor Code, as amended by P.D. No. 1691 so that

LABOR 2 | Atty. Aonuevo | BETT DIGEST GROUP |

33

attorneys fees, if legally payable, can only be charged against


Union funds.
ISSUE: WON the Respondent Counsels legal fees should be
charged against Union funds.
HELD: YES. The attorneys fees should be charged against
Union funds.
RATIO: This is pursuant to Art. 222 (b) 4 of the Labor Code.
Deductions from wages of the employees may only be made
by the employer in cases authorized by law, including
deductions for insurance premiums advanced by the employer
on behalf of the employees as well as union dues where the
right to check-off is authorized in writing by the individual
employee himself.
The provisions are clear. No check-offs from any amounts due
employees may be effected without individual written
authorizations duly signed by the employees specifically
stating the amount, purpose and beneficiary of the deduction.
The required individual authorizations in this case are wanting.
In fact, petitioner employees are vigorously objecting. The
question asked in the plebiscite, besides not being explicit,
assumed that there was no dispute relative to attorneys fees.

4 Art. 222. Appearances and Fees.xxx xxx


(b) No attorneys fees, negotiation fees or similar charges of any kind
arising from any collective bargaining negotiations or conclusion of the
collective bargaining agreement shall be imposed on any individual
member of the contracting union; Provided, however, that attorneys fees
may be charged against union funds in an amount to be agreed upon by
the parties. Any contract, agreement or arrangement of any sort to the
contrary shall be null and void.

Contrary to respondent Unions and Counsels stand, the


benefits awarded to PLDT employees still formed part of the
collective bargaining negotiations although placed already
under compulsory arbitration. This is not the mandatory
activity under the Code which dispenses with individual
written authorizations for check-offs, notwithstanding its
compulsory nature.
CASE 51: Diokno v. Cacdac, TUTAAN
TOPIC: Enforcement and remedies
Facts: The First Line Association of Meralco Supervisory
Employees (FLAMES) is a legitimate labor organization which
is the supervisory union of Meralco. Petitioners and private
respondents are members of FLAMES. The FLAMES
Executive Board created the Committee on Election
(COMELEC) for the conduct of its union elections. The
COMELEC was composed of petitioner Dante M. Tong as its
chairman, and petitioners Jaime C. Mendoza and Romeo M.
Macapulay as members. Subsequently, private respondents
Jimmy S. Ong, Nardito C. Alvarez, Alfredo J. Escall, and Jaime
T. Valeriano filed their respective certificates of candidacy.
The COMELEC rejected Jimmy S. Ongs candidacy on the
ground that he was not a member of FLAMES. Meanwhile,
the certificates of candidacy of Nardito C. Alvarez, Alfredo J.
Escall, and Jaime T. Valeriano were similarly rejected on the
basis of the exclusion of their department from the scope of
the existing collective bargaining agreement (CBA). The
employees assigned to the aforesaid department are allegedly
deemed disqualified from membership in the union for being
confidential employees.

LABOR 2 | Atty. Aonuevo | BETT DIGEST GROUP |

34

Private respondents (Ong et al.) filed a Petition before the


Med-Arbitration Unit of the Department of Labor and
Employment (DOLE). They prayed, inter alia, for the
nullification of the order of the COMELEC which disallowed
their candidacy. They further prayed that petitioners be
directed to render an accounting of funds with full and detailed
disclosure of expenditures and financial transactions; and that
a representative from the Bureau of Labor Relations (BLR) be
designated to act as chairman of the COMELEC in lieu of
petitioner Dante M. Tong. DOLE-NCR Regional Director Alex
E. Maraan issued an Order directing DOLE personnel to
observe the conduct of the FLAMES election.
Petitioners filed a Petition with the COMELEC seeking the
disqualification of private respondents Daya, et al. Petitioners
alleged that Daya, et al., allowed themselves to be assisted by
non-union members, and committed acts of disloyalty which
are inimical to the interest of FLAMES. In their campaign, they
allegedly colluded with the officers of the Meralco Savings and
Loan Association (MESALA) and the Meralco Mutual Aid and
Benefits Association (MEMABA) and exerted undue influence
on the members of FLAMES. The COMELEC issued a
Decision, declaring Daya, et al., officially disqualified to run
and/or to participate in FLAMES elections. The COMELEC
also resolved to exclude their names from the list of
candidates in the polls or precincts, and further declared that
any vote cast in their favor shall not be counted. According to
the COMELEC, Daya, et al., violated Article IV, Section 4(a)(6)
of the FLAMES Constitution and By-Laws (CBL) by allowing
non-members to aid them in their campaign. Their acts of
solicitation for support from non-union members were deemed
inimical to the interest of FLAMES.

Private respondents Daya, et al., along with Ong, et al., filed


with the Med-Arbitration Unit of the DOLE-NCR, a Petition to:
a) Nullify Order of Disqualification; b) Nullify Election
Proceedings and Counting of Votes; c) Declare Failure of
Election; and d) Declare Holding of New Election to be
Controlled and Supervised by the DOLE.
On 14 May 2003, another group, Jimenez, et al. filed a Petition
with the Med-Arbitration Unit of the DOLE-NCR against
petitioners to nullify the 7 May 2003 election on the ground
that the same was not free, orderly, and peaceful. It was
subsequently consolidated with the Petition of Daya, et al. and
the earlier Petition of Ong, et al.
Meanwhile, the records show that a subsequent election was
held on 30 June 2004, which was participated in and won by
herein private respondents Daya, et al. The validity of the 30
June 2004 elections was assailed by herein petitioners before
the DOLE and taken to the Court of Appeals in on certiorari,
which case does not concern us in the instant Petition. The
Court of Appeals, in the aforesaid case, rendered a Decision,
upholding the validity of the 30 June 2004 elections, and the
declaration of herein private respondents Daya, et al., as the
duly elected winners therein.
Med-Arbiter Tranquilino B. Reyes, Jr.: Rendered a Decision
in favor of private respondents, Daya, et al. However, the
petition of Jimenez, et al., was dismissed because it was
premature, it appearing that the COMELEC had not yet
resolved their protest prior to their resort to the Med-Arbiter.
Finally, the Petition of Ong, et al., seeking to declare
themselves as bona fide members of FLAMES was ordered
dismissed.

LABOR 2 | Atty. Aonuevo | BETT DIGEST GROUP |

35

The Med-Arbiter reversed the disqualification imposed by the


COMELEC against private respondents Daya, et al. He said
that the COMELEC accepted all the allegations of petitioners
against private respondents Daya, et al., sans evidence to
substantiate the same.
Moreover, he found that the
COMELEC erred in relying on Article IV, Section 4(a) (6) of the
CBL as basis for their disqualification. The Med-Arbiter read
the aforesaid provision to refer to the dismissal and/or
expulsion of a member from FLAMES, but not to the
disqualification of a member as a candidate in a union
election. He rationalized that the COMELEC cannot disqualify
a candidate on the same grounds for expulsion of members,
which power is vested by the CBL on the Executive Board.
The Med-Arbiter also held that there was a denial of due
process because the COMELEC failed to receive private
respondents Daya, et al.s motion for reconsideration of the
order of their disqualification. The COMELEC was also found
to have refused to receive their written protest in violation of
the unions CBL.
Lastly, the Med-Arbiter defended his jurisdiction over the case.
He concluded that even as the election of union officers is an
internal affair of the union, his office has the right to inquire into
the merits and conduct of the election when its jurisdiction is
sought.
BLR Director: Director of the BLR issued a Resolution,
affirming in toto the assailed Decision of the Med-Arbiter.
CA: affirmed the finding of the BLR Director that the
COMELEC, in disqualifying private respondents Daya, et al.,
committed a procedural shortcut. On the matter of the failure
of private respondents Daya, et al. to come up with 30 percent
(30%) members support in filing the Petition to Nullify the
COMELECs Decision before the Med-Arbiter, the Court of
Appeals said that the petition did not involve the entire

membership of FLAMES, so there was no need to comply with


the aforesaid requirement.
It is the stance of petitioner that Article 226 of the Labor Code
which grants power to the BLR to resolve inter-union and intraunion disputes is dead law, and has been amended by Section
14 of Republic Act No. 6715, whereby the conciliation,
mediation and voluntary arbitration functions of the BLR had
been transferred to the National Conciliation and Mediation
Board.
Issue: WON the BLR has jurisdiction? YES
Ratio: Article 226 of the Labor Code BUREAU OF LABOR
RELATIONS. The Bureau of Labor Relations and the Labor
Relations Divisions in the regional offices of the Department of
Labor shall have original and exclusive authority to act, at their
own initiative or upon request of either or both parties, on all
inter-union and intra-union conflicts, and all disputes,
grievances or problems arising from or affecting labormanagement relations in all workplaces whether agricultural or
nonagricultural, except those arising from the implementation
or interpretation of collective bargaining agreements which
shall be the subject of grievance procedure and/or voluntary
arbitration. The Bureau shall have fifteen (15) working days to
act on labor cases before it, subject to extension by agreement
of the parties.
The amendment to Article 226, as couched in Republic Act No.
6715, which is relied upon by petitioners in arguing that the
BLR had been divested of its jurisdiction, simply reads, thus:

LABOR 2 | Atty. Aonuevo | BETT DIGEST GROUP |

36

Sec. 14. The second paragraph of Article 226 of the


same Code is likewise hereby amended to read as
follows:
"The Bureau shall have fifteen (15) calendar days to
act on labor cases before it, subject to extension by
agreement of the parties."
This Court in Bautista v. Court of Appeals, interpreting Article
226 of the Labor Code, was explicit in declaring that the BLR
has the original and exclusive jurisdiction on all inter-union and
intra-union conflicts. We said that since Article 226 of the
Labor Code has declared that the BLR shall have original and
exclusive authority to act on all inter-union and intra-union
conflicts, there should be no more doubt as to its jurisdiction.
As defined, an intra-union conflict would refer to a conflict
within or inside a labor union, while an inter-union controversy
or dispute is one occurring or carried on between or among
unions.

propriety of the disqualification of private respondents Daya, et


al., by the FLAMES COMELEC in the 7 May 2003 elections. It
must also be stressed that even as the dispute involves
allegations that private respondents Daya, et al., sought the
help of non-members of the union in their election campaign to
the detriment of FLAMES, the same does not detract from the
real character of the controversy. It remains as one which
involves the grievance over the constitution and bylaws of a
union, and it is a controversy involving members of the union.
Moreover, the non-members of the union who were alleged to
have aided private respondents Daya, et al., are not parties in
the case. We are, therefore, unable to understand petitioners
persistence in placing the controversy outside of the
jurisdiction of the BLR. The law is very clear. It requires no
further interpretation. The Petition which was initiated by
private respondents Daya, et al., before the BLR was properly
within its cognizance, it being an intra-union dispute.
Indubitably, when private respondents Daya, et al., brought the
case to the BLR, it was an invocation of the power and
authority of the BLR to act on an intra-union conflict.

The controversy in the case at bar is an intra-union dispute.


There is no question that this is one which involves a dispute
within or inside FLAMES, a labor union. At issue is the

LABOR 2 | Atty. Aonuevo | BETT DIGEST GROUP |

37

Вам также может понравиться